Location via proxy:   [ UP ]  
[Report a bug]   [Manage cookies]                

Aaos Spine 2020.

Download as pdf or txt
Download as pdf or txt
You are on page 1of 71

Adult Spine Scored and Recorded Self-Assessment Examination 2020

Adult Spine Scored and Recorded Self-


Assessment Examination 2020
Adult Spine Scored and Recorded Self-Assessment Examination 2020

Question 1 of 100
Figures 1 and 2 are the MRI scans of the spine of a 20-year-old college football player
who complains of severe right arm pain after making a tackle. He has numbness of
the right thumb and index finger but has 5/5 strength in both arms, and his
neurological examination is otherwise unremarkable. You counsel the patient that he
can return to play when/if

A. nonoperative care has a higher chance than surgery of allowing him to be able to return
to play.
B. he is asymptomatic with normal range of motion and a negative neurological examination.
C. he is not likely to be able to return to football.
D. after surgical decompression.

R: B
The patient has sustained a herniated disk, which is likely causing his radicular symptoms.
The patient does not have significant weakness or myelopathic symptoms, and initial
treatment should be nonoperative. However, Hsu found that surgical treatment is shown to
result in a better chance of returning to play in National Football League players. Regardless
the treatment, the patient should not be allowed to return to play until he is asymptomatic
with normal range of motion and a negative neurological examination.

Question 2 of 100
Long-term outcomes that compare two-level anterior cervical diskectomy and fusion
(ACDF) with two-level cervical disk arthroplasty suggest that

A. two-level ACDF is not an effective procedure for the treatment of cervical radiculopathy
at two adjacent levels.
B. two-level cervical disk arthroplasty has a higher rate of revision surgery compared with
two-level ACDF.
C. two-level cervical disk arthroplasty has a lower rate of radiographic adjacent-level
degeneration.
D. two-level cervical disk arthroplasty has an 80% rate of heterotopic ossification production
at the operated levels.
Adult Spine Scored and Recorded Self-Assessment Examination 2020

R: C
Studies report a radiographic heterotopic ossification (HO) rate of >40% after long-term
follow-up of cervical disk replacements. Therefore, cervical disk replacements do carry a
significant rate of HO development. However, the clinical significance is difficult to determine,
as most of those patients are not symptomatic. Long-term studies show a lower revision rate
and lower radiographic adjacent segment degeneration with two-level cervical disk
replacement compared with two-level ACDF. However, both treatment options are
considered effective procedures for the treatment of cervical radiculopathy at two adjacent
levels.

Question 3 of 100
Figure 1 is the CT scan of a 36-year-old man who fell from a roof. Eight hours later at
the emergency department he describes low-back pain with numbness and weakness
in his bilateral lower extremity. A neurologic examination reveals 2/5 strength in his
quadriceps and iliopsoas bilaterally, 2/5 strength in his right anterior tibialis and
gastrocsoleus, and 1/5 strength in his left anterior tibialis and gastrocsoleus. Two
hours later, strength in his lower extremities has diminished markedly. What is the
best next step?

A. Intravenous (IV) methylprednisolone with a 30-mg/kg loading dose followed by


continuous infusion of 5.4 mg/kg/hour for 24 hours
B. Immediate awake traction reduction
C. Emergent open reduction/decompression
D. Admission to the intensive care unit for fluid resuscitation followed by
reduction/decompression when stable
R: C
Any progressive neurologic deficit requires emergent surgical intervention. Lumbar injuries
cannot be reliably reduced with traction. Although IV steroids and management of mean
arterial blood pressure are appropriate interventions for injuries in the region of the conus
medullaris, steroids are only indicated when given within 8 hours of injury and are not
appropriate as a sole means of management for progressive neurologic deficit.

Question 4 of 100
During a minimally invasive surgery (MIS) to perform a lateral lumbar interbody fusion,
approach to which level carries the highest risk of injury to the lumbar trunk?
Adult Spine Scored and Recorded Self-Assessment Examination 2020

A. L1-L2
B. L2-L3
C. L3-L4
D. L4-L5
R: D
The MIS approach to the L4-L5 level places the lumbar nerve roots at the highest risk of
injury during surgery. The psoas muscle and lumbar nerve roots course more anteriorly as
they move caudally from L1 toward L5. Intraoperative neuromonitoring is considered
standard in these procedures, especially at the L4-L5 to reduce the risk of iatrogenic nerve
injury.

Question 5 of 100
A patient has sustained the injury shown in the MRI scan in Figure 1. He has 3/5
strength in the biceps and 0/5 strength in all muscle groups distally. There is no
sensation to light touch or pin prick from the upper chest and distally.
Bulbocavernosus reflex is present. His perianal sensation is intact to pin prick. What
American Spinal Injury Association (ASIA) grading do you give the injury?

A. A
B. B
C. C
D. D

R: B
The MRI cervical spine scan reveals a C4-C5 traumatic anterolisthesis with severe spinal
stenosis. There is evidence of spinal cord swelling and myelomalacia at this level. A physical
examination is provided. The ASIA scale provides grading of severity of a spinal cord injury.
The grading incorporates strength and motor function based on the spinal level.
Adult Spine Scored and Recorded Self-Assessment Examination 2020

ASIA A is a complete spinal cord injury with no motor or sensory function. ASIA B is an
incomplete spinal cord injury with no motor function below the spinal level of injury and sacral
sparing of sensation. ASIA C is an incomplete spinal cord injury with motor function <3 of 5
in more than half of the muscles below the spinal level of injury. ASIA D is an incomplete
spinal cord injury with motor function >3 of 5 in at least half of the muscles below the spinal
level of injury. ASIA E is normal sensation and motor function.

Question 6 of 100
Which intervention most effectively prevents surgical-site infections following spine
surgery?

A. Bathing the day of surgery


B. Intravenous (IV) vancomycin
C. Preincision IV antibiotics
D. Vancomycin powder in wound

R: C
The use of IV antibiotics for prophylaxis of surgical-site infection is supported by Level 1
evidence in spine surgery. It has been given a &quot;B&quot; recommendation by the North
American Spine Society. The use of specific bathing solutions the day of surgery may be
beneficial, but the evidence in spine surgery is lacking. Similarly, evidence for use of
vancomycin (either topically or IV) is not supported by high-level studies, although
retrospective and basic science studies support topical vancomycin use.

Question 7 of 100
Figures 1 and 2 are the radiograph and MRI scans, respectively, of a 45-year-old
hunter who sustains a fall down a 20-foot ravine, landing on his buttocks. Despite
severe low back pain, he is able to walk out of the woods and call for help. Physical
examination reveals 5/5 motor strength and normal sensation to light touch to lower
extremities. What is the most appropriate additional test/study needed?

A. Radiographs of calanei
Adult Spine Scored and Recorded Self-Assessment Examination 2020

B. CT scan of the head


C. CT scan of the abdomen
D. Rectal examination

R: D
The images reveal a severe burst fracture of L2 with significant middle column bony
retropulsion into the spinal canal. Although additional tests may be required in specific spinal
injuries, a rectal examination is a required part of the spinal injury examination. An L2 burst
fracture can cause injury to the spinal cord at the conus medullaris, which normally
terminates at L1-L2. Typically, such injuries are a mixed cauda equina nerve root injury with
a spinal cord injury of the conus medullaris. In this case, the patient fell onto his buttocks
without hitting his head thus, not requiring a head CT. He is able to walk and does not
complain of foot pain.

Question 8 of 100
A 61-year-old woman has a history of a left thigh melanoma that was widely resected
approximately 12 years ago. The patient also has a history of nephrolithiasis and has
just undergone an abdominal CT scan for evaluation of her kidneys. The patient was
incidentally found to have a lytic lesion of the sacrum. A radiograph is shown in
Figure 1. CT scans are shown in Figures 2 through 4. Open biopsy was performed
and the biopsy specimen is shown in Figure 5. What is the most appropriate
management?

A. En bloc excision of the sacrum and lesion


B. Embolization of the vascular supply to the lesion
C. Chemotherapy as the initial treatment, followed by surgical resection
D. Bisphosphonate therapy

R: D
The patient is asymptomatic from the sacral lesion, but has a history of a malignant lesion,
so the suspicion is high that the sacral lesion is a recurrence or a metastatic lesion. The
radiograph and CT scans show a lytic lesion within the osseous margins of the sacrum, and
the histologic section shows no malignant cells. The diagnosis is Paget's disease, which is
typically treated medically. Bisphosphonate treatment is typical, but is currently controversial
as to whether it helps more than just controlling the local symptoms. Radiographic features
vary but can reveal cortical thickening, coarse trabeculae, and sclerotic or enlarged vertebral
bodies. The sacrum is typically involved. Histologically there is &quot;mosaic&quot;
appearing bone with numerous random intersecting lines, overly active osteoclasts and/or
Adult Spine Scored and Recorded Self-Assessment Examination 2020

osteoblasts, and fibrous tissue replacement of marrow. The specimen shows disordered
appearance of the bone and the multiple intersecting lines.

Question 9 of 100
A 35-year old man has had 8 weeks of progressive midback pain and persistent left
thigh pain. He tried chiropractic manipulation and lumbar traction, which were both
unsuccessful in pain relief. MRI scans reveal a left-sided L2-L3 foraminal disk
herniation. He is subsequently referred to an interventional pain specialist. A left
transforaminal epidural injection is scheduled. During the procedure, the patient
develops rapid bilateral leg weakness and subsequent paraplegia. Post procedure
MRI is shown in Figures 1 and 2. What is the most likely cause of this lesion
neurological injury?

A. Epidural hematoma
B. Intra-arterial injection
C. Intravenous injection
D. L2 nerve puncture

R: B
Complication rates for percutaneous interventional procedures are low (1-2%). Potential
risks for epidural injections include dural injury, cerebrospinal fluid leak, infection, nerve
puncture, intrathecal injection, and intravascular injection. Furman and associates reported
8% incidence of inadvertent vascular puncture from lumbar transforaminal injection. In this
patient, there was injection into an L2 radiculomedullary artery, which ultimately caused
catastrophic spinal cord ischemia and infarction. The dominant radiculomedullary artery,
artery of Adamkiewicz, is the major blood supply for the anterior cord. Adamkiewicz enters
the cord on the left from T9 to L2 level in 85% of people. The MRI scan shown, taken 48
hours after injury, indicates classic cord infarction with hyperintense cord signal on sagittal
film. The axial image also shows hyperintense signal, predominantly in the gray matter with
&quot;owl's eye&quot; pattern. Epidural hematoma would show a high T2 signal extradural
compressive lesion on MRI. Intravenous injections are rarely dangerous. L2 nerve injury
from a puncture would cause unilateral L2 nerve pain (dysesthesia), hypoesthesia, and/or
palsy.
Adult Spine Scored and Recorded Self-Assessment Examination 2020

Question 10 of 100
A 59-year-old woman with a history of gastric bypass 1 year ago and symptomatic L5-
S1 isthmic spondylolisthesis is seen. She has been symptomatic with bilateral leg
pain for 6 months. She has tried physical therapy, selective nerve root injections, and
nonsteroidal anti-inflammatory drugs with minimal relief. She is offered surgical
intervention consisting of L5-S1 posterior spinal and interbody fusion along with a
Gill laminectomy. As part of surgical planning, the surgeon should consider ordering

A. thyroid laboratories.
B. hemoglobin A1c.
C. dual-energy x-ray absorptiometry (DEXA) scan.
D. medial branch block.
R: C
Because of the increasing rates of obesity, gastric bypass surgeries are becoming
increasingly prevalent. Gastric bypass surgery is associated with negative effects on bone
metabolism and can result in decreased bone mass. Some risk factors include changes in
absorption (vitamin D), loss of muscle mass, and hormone changes. In a study of 48 patients
undergoing Roux-en-Y gastric bypass, DEXA 6 months and 12 months postoperative from
gastric bypass noted a 5% and 8% decrease, respectively, in femoral neck bone mineral
density, compared with preoperative density. In patients with osteopenia or osteoporosis
undergoing instrumented spinal fusion, failure of instrumentation may arise and presurgical
planning is required. In patients with a history of gastric bypass undergoing instrumented
spinal fusion, preoperative DEXA scan can diagnose osteopenia and/or osteoporosis and
an appropriate surgical plan can be formulated.

Question 11 of 100
Figures 1 and 2 show CT images from a 24-year-old man who was the unrestrained
driver in a single motor vehicle collision. By report, he was ejected from the vehicle
and initially was found unresponsive. The patient was intubated in the field and then
brought by ambulance to the emergency department, where he was resuscitated
aggressively with crystalloid and blood transfusions. Radiographs were taken and
showed an intracranial hemorrhage, which required emergent burr hole evacuation
by Neurosurgery. In the intensive care unit, his blood pressure is 80/48, and his pulse
is 48. He is breathing spontaneously on the ventilator at 16 breaths per minute. He
can follow commands. Physical examination reveals absent motor function in the
legs, no sensation below the nipple level, and a positive bulbocavernosus reflex. His
skin is warm and dry. What best describes his condition?
Adult Spine Scored and Recorded Self-Assessment Examination 2020

A. Neurogenic shock
B. Spinal shock
C. Hypovolemic shock
D. Septic shock

R: A
This patient has classic neurogenic shock, which usually occurs when a cervical or high
thoracic cord injury disrupts the autonomic pathways and causes a loss of sympathetic tone.
Characteristic hypotension and bradycardia are present due to an unopposed vagal tone.
Low cardiac output also is present, along with venous and arterial dilatation. The treatment
for neurogenic shock is administration of agents called pressors (phenylephrine, dopamine,
dobutamine, and norepinephrine) to improve cardiac contractility and increase peripheral
vascular resistance. Atropine is given to increase the heart rate. Pressors are titrated to keep
the mean arterial pressure above 80 and maintain spinal cord perfusion.

Question 12 of 100
In the MRI scan shown in Figure 1, what is it about this fracture pattern that increases
its risk of nonunion?

A. Early treatment
B. Younger age
C. Posterior displacement >5 mm
D. Angulation <10°

R: C
The patient has sustained a type 2 odontoid fracture. This is a common injury in elderly
patients secondary to a hyperextension injury. The blood supply is tenuous and posterior
displacement has been found to increase the risk of a nonunion.
Adult Spine Scored and Recorded Self-Assessment Examination 2020

Question 13 of 100
Figures 1 through 4 show the radiographs and MRI scans of a 69-year-old woman with
neck and upper extremity pain and progressive deformity of the cervical spine. What
is the most likely diagnosis?

A. Postlaminectomy kyphosis
B. Ankylosing spondylitis
C. Occipitocervical dissociation
D. C3-4 pseudarthrosis

R:A
Laminectomy without fusion for the treatment of cervical spondylotic myelopathy currently
plays a minor role in the management of this disorder because of its many disadvantages.
The actual incidence of postlaminectomy kyphosis is unknown, but is estimated to be
between 11% and 47%. It can result in recurrent myelopathy if the spinal cord becomes
draped over the kyphosis. In addition to the neurologic sequelae, the kyphosis itself can be
a source of neck pain and deformity. Spondylolisthesis can develop, contributing to further
cord compression. In this case, the patient had undergone a previous C4-5 anterior cervical
diskectomy and fusion followed by a posterior laminectomy from C2 through C7, without
fusion. This has resulted in severe kyphosis (i.e. postlaminectomy kyphosis) with grade II-
III spondylolisthesis at C3-4 and a grade I spondylolisthesis at C2-3. While ankylosing
spondylitis can also result in a chin-on-chest deformity secondary to ankylosis, there is no
evidence of marginal syndesmophytes in the imaging studies to suggest this diagnosis. The
occiput is hyperextended on C1 on the lateral upright radiograph to compensate for the
kyphosis in an attempt to maintain horizontal gaze. This results in an unusual appearing
relationship on the imaging studies. However, there is no widening of the distance between
C1 and the occiput and no evidence of soft-tissue injury on the MRI scans to suggest an
acute injury. C3-4 demonstrates an unstable spondylolisthesis and was never intended to
be included in the C4-5 fusion.

Question 14 of 100
Figure 1 is the MRI scan of a patient with cervical spondylotic myelopathy disease.
Considering outcome and complications, a surgeon selecting anterior
decompression approaches to posterior decompression approaches will see
Adult Spine Scored and Recorded Self-Assessment Examination 2020

A. no difference when surgeon discretion used.


B. a higher complication rate.
C. a tendency to treat more diffuse pathology.
D. the approaches are more useful for older patients.

R: A
The AO Spine Classification Group has initiated a number of prospective studies on the
treatment of cervical spondylotic myelopathy. In particular, Fehlings and associates showed
that surgeon choice was important in selecting treatment, because the complications and
outcomes were similar when comparing anterior to posterior approaches. Previously, studies
showed more complications with posterior approaches. Further, anterior approaches are
useful for more focal pathology in younger patients.

Question 15 of 100
Which type of thoracolumbar injury typically involves all three columns, is most
mechanically unstable, and is most associated with complete spinal cord injury?

A. Compression
B. Burst
C. Flexion distraction
D. Translation-rotation
R: D
Translation-rotation injuries typically yield fracture dislocations. This injury pattern involves
the disruption of skeletal and ligamentous elements of the spine to cause a maximum loss
of stability, subsequent deformity in three planes (coronal, axial, and sagittal), and
catastrophic neurologic injury.
Compression injuries occur when a force is applied in flexion and injures the anterior column.
Compression injuries are usually stable and rarely have neurologic sequelae. Burst fractures
occur through axially applied forces, which in turn cause injury to the anterior and middle
columns of the vertebrae at minimum. Neurologic injury can occur through direct
compression of the neural elements by bone fragments or hematoma or by absorption of
Adult Spine Scored and Recorded Self-Assessment Examination 2020

the transferred energy. Flexion distraction injuries typically occur as forces are transmitted
from anterior to posterior, causing injury to the middle and posterior columns.

Question 16 of 100
A 57-year-old man is involved in a rear-ended motor vehicle collision. He is able to
leave his pickup and assist others involved in the collision. The next day he is seen
in the emergency department with low back pain. The patient's radiograph and MRI
scans of the lumbosacral spine are shown in Figures 1 through 3. What is the most
likely diagnosis of his pain?

A. Spinal fluid leakage


B. Lumbosacral degloving
C. Lipomatosis
D. Flexion-distraction injury of the spine

R: B
The patient has sustained a Morel-Lavallee lesion, a degloving injury of the lumbosacral and
pelvic regions. It is sustained by a shear force that tears the subcutaneous tissue off the
underlying muscular fascia. A resulting seroma develops secondary to blood, fat, and
lymphatic fluid. The seroma often needs to be either percutaneously or surgically drained,
depending on the size and associated bony injuries. Risks of inappropriately treated lesions
can result in infection, tissue necrosis, or a chronic seroma.

Question 17 of 100
A 57-year-old man has had a 2-week history of neck pain. He has no history of
radiating symptoms, and has no complaints of numbness or paresthesias. There was
no trauma associated with the onset of the pain. Figure 1 shows the MRI scan initially
obtained by his family physician. What should the patient be told regarding the
prevalence of the MRI findings in his age group?
Adult Spine Scored and Recorded Self-Assessment Examination 2020

A. 20% to 25%
B. 50% to 60%
C. 75% to 80%
D. Greater than 95%

R: B
The MRI findings reveal age-related degenerative changes in the cervical spine, which is a
very common finding in the adult population. Boden and associates evaluated cervical spine
MRI findings on 63 asymptomatic subjects and found that the prevalence of having at least
one degenerative disk was approximately 57% in those older than age 40 years.

Question 18 of 100
The use of demineralized bone matrix (DBM) for posterolateral lumbar fusion has
been shown to

A. have equivalent amounts of bone morphogenetic proteins (BMP) in all DBM preparations.
B. be an effective replacement for local bone.
C. have similar fusion rates to iliac crest bone graft (ICBG) when combined with local bone.
D. produce higher rates of fusion compared with biphasic calcium phosphate.
R: C
The use of DBM has been shown to be an effective bone graft extender when combined
with local bone. Kang and associates performed a side-by-side comparison of ICBG on one
side of a lumbar fusion and local bone and DBM. Both sides showed equivalent fusion rates.
However, the amount of bone morphogenetic proteins available in DBM has been shown to
vary wildly between different preparation of DBM and even within different lots of the same
DBM. This is thought to be related to variability in donors and different companies’ processes
to prepare the DBM. There has been no direct comparison between DBM and calcium
phosphate. DBM has not been directly compared with local bone. It is most commonly used
to extend local bone and not to replace it.

Question 19 of 100
Figures 1 through 3 are the sagittal and axial CT scans and sagittal T2 MR image of a
21-year-old man who was thrown from his motocross bike earlier in the day. He now
Adult Spine Scored and Recorded Self-Assessment Examination 2020

has significant low-back pain; however, he is neurologically intact and has no trouble
voiding urine. A standing plain radiograph obtained the next day is shown in Figure
4. Treatment should involve

A. resumption of full activity as soon as tolerated.


B. a brace.
C. anterior stabilization and fusion.
D. posterior stabilization and fusion.

R: B
Disruption of the posterior ligamentous complex is an important determinant of the stability
of a burst fracture. This patient is neurologically intact and his MR images do not reveal
posterior ligamentous complex (PLC) disruption. The standing radiograph confirms that
overall alignment is acceptably and relatively preserved. Nonsurgical treatment with or
without a brace is acceptable in this scenario; however, the patient should not be cleared to
resume full activity until fracture healing, which may be as long as 3 months after the date
of injury. Anterior or posterior surgery should be reserved for patients with PLC disruption,
neurological injury, or, in some cases, multiple trauma.

Question 20 of 100
Which radiographic parameter is fixed as an adult (Figures 1 through 4)?

A. Figure 1
Adult Spine Scored and Recorded Self-Assessment Examination 2020

B. Figure 2
C. Figure 3
D. Figure 4

R: C
Pelvic incidence is a fixed sagittal parameter in adults. Figure 3 represents the pelvic
incidence. Figure 1 represents the pelvic tilt. Figure 2 is an indirect way of measuring the
sacral slope, as sacral slope = pelvic incidence - pelvic tilt. Figure 4 represents the sacral
slope.

Question 21 of 100
Figure 1 is an axial MRI at the L4-5 level obtained from a 62-year-old man with a 6-
month history of severe right leg pain and weakness in the ankle dorsiflexors. He has
numbness along the medial ankle and dorsolateral aspect of his foot. The structure
identified by the arrow is compressing what neural structure?

A. Traversing right L4 nerve root


B. Exiting right L5 nerve root
C. Exiting left L4 nerve root
D. Exiting right L4 nerve root

R: D
This patient has the clinical symptoms of a right L4 lumbar radiculopathy. The MRI taken at
L4-5 shows a far-lateral/foraminal disk herniation. This disk herniation would compress the
exiting L4 nerve root along with its dorsal root ganglion. The traversing right L4 nerve root
would be seen best in an axial MRI at the L3-4 level. The exiting right L5 nerve root would
be seen best in an axial MRI at the L5-S1 level. The disk herniation in question is right sided.
The left neuroforamen is free in the axial MRI.

Question 22 of 100
During the workup of her hearing loss, a 21-year-old woman had imaging which lead
to further imaging of her spine shown in Figures 1 and 2. She was also noted to be
anemic. Her mother died when she was young, and she was told her grandmother
had a “bone disease”. Her endocrinologist has diagnosed her with Albers-Schonberg
disease. What is the cause of her disease?
Adult Spine Scored and Recorded Self-Assessment Examination 2020

A. Mutations in the chloride 7 (CLCN7) gene


B. Mutations in cathepsin K (CTSK) gene
C. Consumption of lead
D. Mutations in tissue non-specific alkaline phosphatase (TNSALP)

R: A
The patient has autosomal dominant osteopetrosis type II, which is also known as Albers-
Schonberg disease. It can be associated with sclerosis of the skull base, leading to cranial
nerve dysfunction such as hearing loss. It is also associated with marrow replacement
leading to anemia and can be associated with fractures. The images show increased bone
density, and osteopetrosis type II can be associated with a “bone within a bone” type
appearance. CTSK mutations are associated with pyknodysostosis, and TNSALP is
associated with hypophosphatasia. Lead poisoning would not present with these findings.

Question 23 of 100
Figures 1 through 3 represent the MRI scans from a 28-year-old man who was ejected
from a car and sustained a cervical spine injury. He has no motor or sensory function
below C5. In comparison to spinal cord injury without facet dislocation, jumped facets
are associated with
Adult Spine Scored and Recorded Self-Assessment Examination 2020

A. a worse clinical outcome.


B. no impact on the clinical outcome.
C. higher rates of revision surgery.
D. an incomplete resolution of symptoms.

R: A
Cervical jumped facets are severe injuries often associated with permanent neurologic
deficits. In a series of 421 patients with cervical spine injuries enrolled in a multicenter
prospective study, 135 patients (32%) had facet dislocations. Compared with the group
without dislocations, the facet dislocation group had worse neurologic deficits on
presentation and less motor recovery at 1-year follow-up.

Question 24 of 100
A 45-year-old man has had 3 months of increasing upper back pain, “balance" issues,
and heaviness in his legs. His physical examination reveals a normal neurological
examination, but he is noted to be anemic. His MRI scans and biopsy specimen are
shown in Figures 1 through 4. He is determined to have a Spinal Instability Neoplastic
Score (SINS) of 6. After biopsy, what is the most appropriate treatment at this time?

A. Laminectomy
B. Laminectomy with posterior spinal fusion
C. Enbloc vertebral resection and reconstruction
D. Radiation of the lesion

R: D
The MRI scan reveals a large posterior element tumor, which is compressing the spinal cord.
Multiple lesions within the spinal column are consistent with multiple myeloma. Myeloma is
a radiosensitive tumor. Additionally, he has a SINS of 6. This score helps the treating
physician determine the tumor-related instability of the vertebral column to guide the
decision for operative management. A SINS of 0-6 is thought to be stable; 7-12, potentially
stable, and >13, unstable. Appropriate treatment in a neurologically intact patient with a
radiosensitive tumor with a low SINS would be radiation treatment versus surgical treatment,
despite the degree of spinal cord compression.
Adult Spine Scored and Recorded Self-Assessment Examination 2020

Question 25 of 100
A 35-year-old man who has had a 6-month history of low back pain and tenderness
now reports worsening pain and stiffness in the hips and entire back. An AP
radiograph of the pelvis demonstrates fusion of the sacroiliac joints bilaterally. What
is the next most appropriate step in management?

A. Anesthetic injections in both sacroiliac joints


B. Sacroiliac fusion with plate fixation
C. Anti-inflammatory medications, physical therapy, and HLA-B27 testing
D. Immediate bilateral sacroiliac joint aspiration and culture

R: C
The patient has a classic presentation of early ankylosing spondylitis. Sacroiliac joint fusion
is the earliest radiographic finding and is typically followed by cephalad spinal progression.
Early treatment of ankylosing spondylitis consists of nonsteroidal anti-inflammatory drugs
and physical therapy to preserve spinal motion. HLA-B27 testing is positive in most (about
95%) patients; however, it is not pathognomonic because it can be positive with other
conditions. Considering the progressive nature of this disease, further work-up in a patient
with potential ankylosing spondylitis is not warranted. Sacroiliac joint anesthetic injections
and sacroiliac fusion are not recommended treatments for early ankylosing spondylitis.
Aspiration of the sacroiliac joints can be done if sacroiliac joint infection is suspected;
however, in the absence of fever or other constitutional symptoms, infection is unlikely.

Question 26 of 100
Figures 1 and 2 are the radiographs of a 75-year-old patient who has a 1-year history
of progressive low back pain. He reports difficulty ambulating, inability to sit for
extended periods, and pain when arising from a seated position. His medical history
is positive for coronary artery disease, type II diabetes, depression, and mild obesity
(BMI 32). His surgical history is positive for a lumbar fusion 3 years previously.
Laboratory studies show normal CBC and metabolic profile. HgbA1C is 6.3. What
factor is most predictive of his perceived clinical outcome after revision surgery?
Adult Spine Scored and Recorded Self-Assessment Examination 2020

A. Intraoperative cervical spine fluid leak


B. Postoperative infection
C. Depression status
D. Intraoperative blood loss requiring transfusion

R: C
Patient satisfaction ratings are increasingly viewed as important parameters for functional
outcomes, as well as in delivering quality care. Psychosocial influence, however, plays a
paramount role in perceived outcomes. Affective disorders like depression have a highly
significant negative effect on patient-related outcomes and self-interpretation of health
status. Verla and associates reported outcomes after fusion surgery and used SF-36, Visual
Analog Scale, and Oswestry Disability Index scores before and at 1 and 2 years,
postoperatively. They also specifically looked at patients who sustained complications
(major and minor). The results showed no lasting effects from complications on patients'
overall interpretation of health status.

Question 27 of 100
Figures 1 and 2 are the lumbar spine radiographs of a 72-year-old man with no
significant medical history who has had severe back pain for 3 weeks. He denies
radiating symptoms, weakness, or numbness when he is seen in the emergency
department. He is sent home with a soft corset. At his follow-up visit he continues to
describe significant back pain with activity that is not relieved with oral narcotic
mediations. A follow-up CT scan shows a nondisplaced fracture through all 3
columns of the spine. What is the most appropriate treatment?

A. Nonsteroidal anti-inflammatory drugs (SAIDS), physical therapy, and activity modification


B. Continued soft corset use
C. Thoracolumbosacral orthosis (TLSO) bracing
D. Posterior stabilization

R: D
Adult Spine Scored and Recorded Self-Assessment Examination 2020

The patient has previously undiagnosed ankylosing spondylitis. Radiographs reveal


nonmarginal syndesmophytes throughout the lumbar spine. The CT scan reveals a
nondisplaced 3-column fracture. Many patients with missed spinal injuries present in
followup with neurologic worsening or progressive deformity. Fractures can often involve all
3 columns, including the posterior elements. In patients with ankylosing spondylitis, this
represents an unstable injury and a high likelihood of displacement with nonsurgical
treatment. Surgical treatment in the form of a posterior spinal fusion is indicated. Because
the fracture is nondisplaced and the patient is neurologically intact, decompression via an
anterior approach is not indicated. Bracing either with a TLSO brace or a soft corset will not
provide sufficient stability for this fracture pattern. Physical therapy and NSAIDS are not
indicated in this scenario.

Question 28 of 100
A 42-year-old woman has a 3-week history of acute lower back pain with radiation into
the left lower extremity. There is no history of trauma and no systemic symptoms are
noted. Examination reveals a positive straight leg test at 25 degrees on the left side.
Motor testing reveals mild weakness of the gluteus maximus and weakness of the
gastrocnemius at 3/5. Sensory examination reveals decreased sensation along the
lateral aspect of the foot. Knee reflex is intact; however, the ankle reflex is absent.
MRI scans show a posterolateral disk herniation. The diagnosis at this time is
consistent with a herniated nucleus pulposus at what level?

A. L2-3
B. L3-4
C. L4-5
D. L5-S1

R: D
The patient's history and examination findings are consistent with a lumbar disk herniation
at the L5-S1 level. Weakness of the gastrocnemius and gluteus maximus are consistent with
an S1 lumbar radiculopathy. Nerve root tension signs are also consistent with a disk
herniation at L5-S1, which typically affects the traversing S1 nerve root.

Question 29 of 100
Figures 1 and 2 are CT scans obtained from a 68-year-old man who has had
progressive neck pain and stiffness, worsening gait imbalance, upper extremity
weakness, early muscle fatigue, difficulty with fine motor control, and difficulty with
activities of daily living over the past few years. On physical examination, he has a
wide based stiff legged gait, generalized upper extremity weakness, dense sensory
loss in the upper and lower extremities, and markedly brisk reflexes. What is the most
appropriate treatment for this patient?
Adult Spine Scored and Recorded Self-Assessment Examination 2020

A. Observation
B. Cervical epidural injections
C. Multilevel anterior cervical decompression and fusion
D. Posterior cervical laminoplasties from C3-6

R: D
This patient has progressive myelopathy secondary to ossification of the posterior
longitudinal ligament. Diagnostic imaging reveals multilevel cervical cord compression from
C4-6. The patient has maintained reasonable cervical lordosis. A posterior procedure such
as multilevel laminoplasty decompresses the spine, is motion preserving, and has a low
complication rate. Observation and cervical epidural injections are not viable options in
patients with progressive myelopathy. Anterior cervical decompression, including
corpectomy, is an option; however, anterior procedures have an increased risk of
complications such as dural tear or cerebrospinal fluid leak. The axial CT image shows a
"double layer" sign, which is consistent with dural ossification and increases the risk of dural
injury with anterior decompression.

Question 30 of 100
A 28-year-old woman is having low back pain that wakes her up at night. A CT scan
reveals a lytic lesion in the fifth lumbar vertebrae shown in Figure 1. A full staging
workup has not revealed any other sites of disease. She is otherwise healthy, and she
does not smoke. A CT biopsy is shown in Figure 2. She wants to avoid surgery for
this problem if possible and inquires about nonsurgical approaches. How should you
counsel her?
Adult Spine Scored and Recorded Self-Assessment Examination 2020

A. Photon radiation is recommended.


B. Denosumab is an effective treatment.
C. En bloc resection is the best option.
D. Bisphosphonates are effective.

R: B
The patient has a giant cell tumor. Surgery remains the standard of care; however, the
monoclonal antibody against RANKL has been shown to be effective in preventing tumor
progression, and it is an effective nonsurgical option. Radiation is not recommended, as this
is a benign tumor and the patient is young. En bloc resection has been shown to be effective,
but the patient is hoping to avoid surgery. Bisphosphonates are not an effective treatment
for giant cell tumors.

Question 31 of 100
A 23-year-old man is evaluated in the emergency department after a diving accident.
Radiographs reveal bilateral jumped facets at C6-7. Examination reveals no motor
function below the C7 level. There is some maintained sensation in the lower
extremities. What is the patient's current grade on the ASIA (American Spinal Injury
Association) impairment scale?
A. ASIA A
B. ASIA B
C. ASIA C
D. ASIA D

R: B
The American Spinal Injury Association (ASIA) provides a standard method of measurement
of spinal cord injury. The ASIA impairment scale is based on a comprehensive motor and
sensory examination. An ASIA A grade is ascribed to a patient with an injury with no motor
or sensory preservation below the injury. An ASIA B grade is defined as no motor
preservation below the level of injury but some sensory preservation below the injury level.
An ASIA C grade is defined as a motor function grade of less than 3 below the injury level.
An ASIA D grade is defined as a motor function grade of greater than 3 below the injury level.
An ASIA E grade is defined as a normal neurologic examination.

Question 32 of 100
A 23-year-old woman was a restrained driver in a motor vehicle collision yesterday.
She develops neck pain and goes to her primary care physician due to no
improvement in degree of pain. She has no neurological deficits or radicular arm pain.
Workup is negative for fracture. What is the best treatment for her injury shown in
Figure 1?
Adult Spine Scored and Recorded Self-Assessment Examination 2020

A. Injection therapy
B. Regimented physical therapy
C. Soft collar
D. Early mobilization

R: D
The patient has sustained a whiplash injury, which is a soft-tissue injury to the cervical spine.
Her radiographs reveal loss of cervical lordosis secondary to muscle spasm. Various
treatment options have been studied, ranging from aggressive physical therapy to
immobilization. Early mobilization has been shown to provide the best treatment.

Question 33 of 100
Figures 1 and 2 are the radiographs of a 21-year-old man who has a long history of
thoracic back pain. His lumbar spine is asymptomatic. He has failed prolonged
nonsurgical treatment. Surgical correction should consist of

A. anterior release and fusion at T4-10.


B. posterior instrumentation and fusion at T9-L5.
C. posterior instrumentation and fusion at T2-L2.
D. posterior instrumentation and fusion at T6-pelvis.
Adult Spine Scored and Recorded Self-Assessment Examination 2020

R: C
When planning surgical intervention for Scheuermann kyphosis, it is imperative that the
instrumentation and fusion extend across the entirety of the deformity. Distally, this means
extending across the first lordotic disk space. In this scenario, this disk is the L1-L2 disk,
which means the fusion needs to extend to L2. Shorter and longer fusions are not necessary
or appropriate.

Question 34 of 100
A 73-year-old man goes to the emergency department after tripping and falling down
roughly thirteen steps at home. Prior to the injury, the patient had well-controlled
medical comorbidities and was independent with all activities of daily living. Figures
1 through 3 show the injury sustained by the patient. What is the most appropriate
definitive treatment for this patient?

A. Rigid cervical collar


B. Halo vest
C. Posterior C1-C2 instrumentation and fusion
D. Anterior C2 fixation

R: C
The patient has a C1 burst fracture, as well as a grossly displaced C2 fracture. Surgical
treatment should be considered for this patient who has good baseline function and well-
controlled medical comorbidities. A cervical collar would not offer adequate stabilization for
this fracture. Anterior reduction of this C2 fracture would be difficult, and screw fixation of C2
would not address the C1-C2 instability. A halo vest is considered a relative contraindication
in the older patient population. Therefore, posterior C1-C2 fixation is the most appropriate
choice.

Question 35 of 100
A 42-year-old woman reports neck stiffness, upper extremity pain, clumsiness,
weakness, and instability of gait. Examination reveals 4+ of 5 strength in the upper
extremities and 3+ biceps, brachioradialis, and patellar reflexes with a positive
Adult Spine Scored and Recorded Self-Assessment Examination 2020

Hoffman sign bilaterally. MRI and CT scans are shown in Figures 1 and 2. Based on
the history and imaging findings, what is the most likely diagnosis?

A. Diffuse idiopathic skeletal hyperostosis


B. Ankylosing spondylitis
C. Ossification of the posterior longitudinal ligament
D. Rheumatoid arthritis

R: C
The sagittal T2-weighted MRI scan shows moderate-severe multilevel cervical stenosis. The
cord compression is noted to be not only at the disk levels but also at the midvertebral body
levels, and the posterior longitudinal ligament appears to be thickened. The CT scan
confirms that the posterior longitudinal ligament is indeed thickened and ossified, compatible
with a diagnosis of ossification of the posterior longitudinal ligament. This diagnosis is most
common in individuals of Japanese descent and has a genetic linkage. The anterior
osteophytes are smaller than those seen in diffuse idiopathic skeletal hyperostosis and are
not syndesmotic. Patients with ankylosing spondylitis typically have non-marginal
syndesmophytes. Patients with rheumatoid arthritis may have evidence of instability at C1-
C2 on flexion-extension radiographs and subaxial subluxations.

Question 36 of 100
Figures 1 through 7 are the radiograph, MRI, and CT scans of a 21-year-old
developmentally delayed woman who complains of urinary urgency, low back pain,
and gait disturbance. What is the most appropriate treatment at this time?
Adult Spine Scored and Recorded Self-Assessment Examination 2020

A. Laminectomy and spinal fusion

B. Gill laminectomy
C. Transforaminal interbody fusion with percutaneous screws
D. Physical therapy and continued observation

R: A
The patient has a low-grade but high-dysplastic spondylolisthesis (vertical and domed
sacrum) with severe spinal canal stenosis. The MRI scan shows the dysplastic sacrum and
severe central stenosis associated with an intact pars interarticularis, bulging L5-S1 disk,
and domed posterior sacrum. Although many treatments are available for low-grade isthmic
spondylolisthesis, this spondylolisthesis condition requires a complete laminectomy and
possible sacral dome resection because of the severe central stenosis with an intact pars
interarticularis (no lysis) in a patient with early neurological signs (Figures 6 and 7 are CT
scans of the L5 pars without evidence of a lysis). Patients with dysplastic spondylolisthesis
without a lysis can develop cauda equina syndrome with loss of bowel/ bladder function and
weakness of the gastrocsoleus muscles (sacral nerve roots) and should be recognized and
treated with appropriate laminectomy decompression followed by spinal fusion, typically with
posterior instrumentation and interbody fusion. A “Gill” laminectomy is described as removal
of the lamina from pars interarticularis lysis and including the abnormal inferior facets. There
is no lysis in this patient, and while laminectomy is needed, a Gill laminectomy is not possible.
Transforaminal interbody fusion and percutaneous instrumentation does not address the
central spinal stenosis.

Question 37 of 100
Adult Spine Scored and Recorded Self-Assessment Examination 2020

Figures 1 and 2 are the MRIs obtained from a 58-year-old woman who has symptoms
of neurogenic claudication. You elect to treat the patient with a lateral lumbar
interbody fusion with posterior pedicle screw instrumentation but no direct neural
decompression. When deciding on this treatment option, you consider that

A. a 20% chance exists that the indirect decompression alone will not be sufficient and that
a decompression procedure may be needed in the future.
B. indirect decompression results in a 10% to 20% increase in the spinal canal area.
C. placement of the cage closer to the midpoint of the disk (in the sagittal plane) will result
in a larger increase in the spinal canal area.
D. indirect decompression without direct decompression has been shown to produce better
postoperative functional outcome scores.

R: B
In degenerative spondylolisthesis, indirect decompression of the spinal canal has been
shown to be an effective treatment option. Malham and associates conducted a prospective
study of 122 patients and reported an unplanned return to the operating room in 11 patients
(9%). When reviewing these cases retrospectively, the authors felt that failure of indirect
decompression should have been anticipated based on radiographic findings in 10 of these
11 patients who had high-grade, unstable spondylolisthesis or substantial bony lateral
recess stenosis. Sato and associates reported an increase in the spinal canal area of 20%,
whereas Castellvi and associates reported only a 9% increase. Park and associates
reported that positioning the cage within the anterior one-third of disk space is better for
achieving the restoration of the segmental angle without compromising the indirect neural
decompression, if the cage was high enough.

Question 38 of 100
Figures 1 and 2 are the MRI and CT scans, respectively, of a 35-year-old woman with
neck pain and decreased range of motion. What condition is associated with this
syndrome?
Adult Spine Scored and Recorded Self-Assessment Examination 2020

A. Marfan syndrome
B. Loyes-Dietz syndrome
C. Neurofibromatosis
D. Sprengel’s Deformity

R: D
MRI and CT scans of the cervical spine reveal extensive fusion of the cervical spine. This
congenital cervical fusion is often associated with Klippel-Feil syndrome. Klippel-Feil
syndrome is characterized by congenital fusion of the cervical spine, decreased range of
motion of the cervical spine, and low posterior hairline. Sprengel’s deformity (high-riding
scapula) can be found in up to 16.7% of patients with Klippel-Feil syndrome. Patients with
Klippel-Feil syndrome may develop cervical scoliosis and cervical stenosis at the adjacent
unfused level. The prevalence of congenital cervical fusion is reported to be 1 in 172.

Question 39 of 100
A 69-year-old patient with diabetes has had acute-onset back pain and difficulty with
ambulation for several hours. Evaluation reveals a temperature of 38.3°C, a white
blood cell (WBC) count of 14000/μL (reference range [rr], 4500-11000/μL), C-reactive
protein (CRP) level of 120 mg/L (rr, 0.08-3.1 mg/L), erythrocyte sedimentation rate of
130 mm/h (rr, 0-20 mm/h), normal rectal examination findings, and normal sensation
to light touch. Motor function testing of the lower extremities reveals 3/5 ankle
dorsiflexion and 4/5 plantar flexion strength bilaterally. An MR image reveals a large
epidural abscess from L1-5. What is the most appropriate treatment at this time?

A. Medical management with intravenous (IV) antibiotics and observation


B. CT-guided aspiration of the abscess before initiating antibiotics
C. Surgical decompression and IV antibiotics
Adult Spine Scored and Recorded Self-Assessment Examination 2020

D. Blood cultures and re-evaluation in 24 hours

R: C
Epidural abscess is a serious and potentially disastrous condition. Although medical
management is effective in some situations, surgical decompression is considered urgent
with the presence of a neurological deficit. Medical management can be considered in the
case of a neurologically intact patient, particularly when the microorganism has been
identified. If medical management is chosen, careful observation and serial examination for
neurologic deterioration is required. Surgical decompression is indicated if a patient's
neurologic status worsens or if medical management failure is noted. Additionally, diabetes,
a CRP level higher than 115 mg/L, WBC higher than 12500/μL , and bacteremia have proven
predictive of medical treatment failure. This patient would be a better candidate for urgent
surgical decompression and subsequent IV antibiotics than for medical management.

Question 40 of 100
A 28-year-old Hispanic male assembly line worker sustains an injury while lifting a
40-lb bag onto a palette. He experiences immediate low back pain, and within 5 days,
he develops severe left leg pain. His MRI scans are shown in Figures 1 and 2. What
factor is the most predictive in proceeding with surgery?

A. Ethnicity
B. Age
C. Physician specialty
D. Gender
Adult Spine Scored and Recorded Self-Assessment Examination 2020

R: C

Workers’ compensation is a system that provides healthcare and wage-replacement


benefits for workers injured in the occupational setting. Back pain is the most common
workers compensation claim in the United States, accounting for up to 25% of all claims and
one-third of total compensation costs. Numerous studies have reported that workers’
compensation is an independent negative risk factor for unsatisfactory outcomes after
surgical procedures.

Keeney and associates published a prospective study looking at which factors were
predictive for proceeding to surgery in the workers’ compensation population. Their findings
showed that young age (<35 years-old), female gender, and Hispanic ethnicity were
negative predictive factors for proceeding with surgical treatment. Which medical
professional the work compensation patient sought made a difference; nearly 43% of injured
workers whose first visit was to a surgeon eventually underwent a surgical procedure.
Question 41 of 100
Figure 1 is the sagittal MR image of a 56-year-old woman who has a 3-year history of
severe back pain. Her pain is worse with flexion at the lumbosacral junction and is
relieved with extension. She denies any pain in her lower extremities and has no
symptoms of neurogenic claudication. Which mediators play roles in the
pathogenesis of this condition?

A. Transforming growth factor-beta (TGF-β), bone morphogenetic protein-2 (BMP-2), latent


membrane protein 1
B. Tissue inhibitor of matrix metallo-proteinase-1 (MMP-1), growth and development factor-
5, noggin
C. Gremlin, MMP, biglycan
D. Tumor necrosis factor-alpha (TNF-α), Interleukin-1 (IL-1), MMP

R: D
Adult Spine Scored and Recorded Self-Assessment Examination 2020

The patient has degenerative disk disease with diskogenic back pain. Several studies in
both humans and animals have implicated TNF-α, IL-1, and MMP in extracellular matrix
degeneration and disk degradation. TGF-β, BMP-2, latent membrane protein 1, and growth
and development factor-5 are all postulated to play anabolic roles in the intervertebral disk.
Biglycan is a small leucine-rich proteoglycan that regulates extracellular matrix assembly
within the disk. Noggin and gremlin are biochemical factors not involved in disk degradation.

Question 42 of 100
Percutaneous pedicle screw fixation can be used in the surgical treatment of adult
patients with certain thoracolumbar spine fractures. A limitation of the use of
percutaneous pedicle screw fixation for thoracolumbar trauma includes

A. facet dislocations.
B. bony chance fractures.
C. thoracolumbar burst fractures without neurological compromise.
D. extension type fractures in ankylosing spondylitis.

R: A
Use of percutaneous pedicle screw fixation has been advocated in patients with chance
fractures, thoracolumbar burst fractures without neurological compromise, and extension-
type fractures in ankylosing spondylitis. However, percutaneous fixation is not advocated for
facet dislocations, which usually require open reduction prior to fixation.
Question 43 of 100
Advanced imaging, to include MRI and CT, have been obtained in the workup of
patients with low back pain. What imaging finding has been associated with reasons
for back pain?

A. Disk degeneration
B. Facet arthropathy
C. Spinal stenosis
D. Spondylolysis

R: C
Low back pain remains a common presenting condition to not only primary care physicians,
but to subspecialists. Studies assessing the anatomy of the spine, to include the
intervertebral disks, vertebral body morphology, facet joints, and the paraspinal muscles
have been performed. Spinal stenosis is the only advanced imaging finding that has been
associated with reproducible reasons for back pain.

Question 44 of 100
Adult Spine Scored and Recorded Self-Assessment Examination 2020

Figures 1 and 2 show the standing posteroanterior and lateral radiographs of a 59-
year-old woman with adult idiopathic scoliosis. She underwent a prior decompressive
laminectomy and fusion at L4-S1 to address lumbar stenosis. She now reports
progressive lower back pain and a feeling of being shifted to the right. If surgical
intervention is considered, what is the most important goal in improving her health-
related quality of life (HRQL) outcomes?

A. Correction of the thoracolumbar curve


B. Sagittal balance
C. Coronal balance
D. Shoulder balance

R: B
Sagittal balance is the most reliable predictor of clinical symptoms and HRQL outcomes on
the SRS 29, SF-12, and Oswestry Disability Index. Coronal balance, shoulder balance,
curve magnitude, and degree of curve correction are less critical in determining clinical
symptoms and outcomes.

Question 45 of 100
A 72-year-old woman with a history of lumbar fusion has developed a fixed sagittal
deformity for which she compensates with pelvic retroversion. She is unable to walk
more than 20 feet because of pain, and she is interested in having a corrective surgery
for this problem. She has been told that she will require a three-column osteotomy,
along with long posterior thoracolumbar fusion. She has renal disease for which her
medication must be adjusted to prevent further renal insufficiency. In addition, she
has a remote history of popliteal venous thrombosis after a long flight. She is
concerned about her risk of postoperative venous thromboembolism (VTE). How
would you counsel her regarding the postoperative VTE?
Adult Spine Scored and Recorded Self-Assessment Examination 2020

A. The risks of chemical anticoagulation outweigh the benefits in adult spinal surgery.
B. An inferior vena cava (IVC) filter is considered the standard of care for VTE prevention in
the spine.
C. She has several risk factors for postoperative VTE.
D. Early mobilization will completely mitigate her risk for VTE.

R: C
The patient has several risk factors for postoperative VTE including older age, previous VTE,
renal disease, and expected long duration of surgery to correct her deformity. VTE is
uncommon after spinal surgery, but each patient must be evaluated individually, and this
patient is at higher risk. It is unknown if the risks of chemical anticoagulation outweigh the
benefits in this patient. IVC filters may be useful, but they are not considered the standard
of care. Early mobilization will certainly help prevent VTE, but it will not completely mitigate
her risk.

Question 46 of 100
Figure 1 is the radiograph of a 15-year-old boy with scoliosis. He has back pain and
spinal asymmetry. Examination reveals a spinal curvature without cutaneous
manifestations. Neurological examination reveals a normal motor and sensory
examination, normal deep tendon reflexes, present superficial abdominal reflexes,
and negative Babinski sign. His MRI scans are shown in Figures 2 and 3. What is the
most appropriate treatment at this time?

A. Brace and continued observation


B. Cerebral spinal fluid shunt
C. Posterior fossa decompression
D. Posterior thoracic spinal fusion

R: D
Adult Spine Scored and Recorded Self-Assessment Examination 2020

The MRI scans reveal a spinal cord with a noted central spinal canal syrinx. The patient has
a normal neurological examination. There is no evidence of Chiari malformation or tethered
spinal cord. Thus, for this patient, a neurosurgical evaluation is not required nor is a cerebral
spinal fluid shunt. As the deformity has progressed past 50° in a skeletally immature
teenager, brace treatment is no longer appropriate, and surgical correction of the scoliosis
is the most appropriate treatment.
Question 47 of 100
A 69-year-old man has nonpainful weakness in the upper and lower extremities. He
also notes progressive instability in his gait and increasing difficulty ambulating, as
well as manipulating small objects with his hands. MRI scans of his cervical spine are
shown in Figures 1 and 2. When would be the most appropriate time to proceed with
surgical treatment?

A. When the patient is medically stable for surgery


B. When the MRI scans show multisegmental high-intensity intramedullary signal changes
on T2-weighted sequences
C. When he reaches a Nurick grade of IV for his preoperative neurologic function
D. When he reports neck and/or extremity pain that becomes intolerable or not controlled
by medication

R: A
The natural history of cervical myelopathy is one of slow deterioration over time, typically in
a stepwise fashion with a variable period of stable neurologic function. More recent studies
suggest that surgery should be performed as soon as possible when cervical spondylotic
myelopathy has been diagnosed. Both anterior and posterior are effective and there is no
statistical difference between their outcomes. Surgical outcome is related to the patient's
age, disease course, the presence of osseous spinal stenosis, preoperative comorbidities,
the preoperative spinal cord functional score, and the presence of high-signal abnormalities
on T2-weighted images. To improve the operative result, all the influencing factors should
be considered. Patients with focal high-intensity intramedullary signal changes on T2-
weighted images have better clinical outcomes following surgery than do patients with
demonstrable multisegmental high-intensity intramedullary signal changes on T2-weighted
sequences. The transverse area and shape of the spinal cord at the involved segment may
Adult Spine Scored and Recorded Self-Assessment Examination 2020

also be predictive of surgical outcome. With progressive compression, the cross section of
the spinal cord changes from a boomerang shape to a teardrop shape to a triangular shape.
In patients with a Nurick grade of I, there are signs of cord involvement, but gait remains
normal. With a Nurick grade of II, there are mild gait abnormalities, not affecting the patient's
employment status. With a Nurick grade of III, gait abnormalities prevent employment, but
the patient remains able to ambulate without assistance. In Nurick grade IV, the patient is
only able to ambulate with assistance. In Nurick grade V, the patient is chair-bound or
bedridden. Clearly, it is desirable to operate when the patient is functioning with a Nurick
grade of I or II. Whereas many patients presenting with cervical spondylotic myelopathy also
report axial neck pain and radicular symptoms in the upper extremities, this is not always
the case. Surgical intervention will generally be effective in eliminating this pain; however,
the pain is not the determining factor for performing surgery. Surgery is performed to
preserve and restore function.

Question 48 of 100
Figure 1 is the MRI scan of a 67-year-old woman with ataxic gait and decreased hand
dexterity. What has been implicated as being part of the pathophysiologic process
leading to the patient’s symptoms?

A. Increased number of endothelial cells lining the small blood vessels surrounding the
spinal cord
B. Spinal cord ischemia
C. Apoptosis of neurons and oligodendrocytes
D. Decreased expression of interleukin 1 (IL-1) and matrix metalloproteinases (MMP)

R: B
The patient has symptoms and radiographic findings consistent with the diagnosis of
myelopathy. Spinal cord ischemia has long been theorized to be the mechanism behind the
development of myelopathy. However, recent experimental models have some no to minimal
decrease is blood flow in cases of moderate myelopathy. Ischemia typically causes cell
death by necrosis; however, human and animal studies have demonstrated that cell death
in the setting of myelopathy is regulated through cell apoptosis. Vascular endothelial cells
Adult Spine Scored and Recorded Self-Assessment Examination 2020

have been shown to decrease in number in the setting of myelopathy. IL-1 and MMP
expression is increased in the setting of inflammation.

Question 49 of 100
Figures 1 through 3 show sagittal and axial MRIs and a radiograph from a 77-year-old
woman with leg pain when standing and walking of 1 year duration. The pain improves
when she leans forward. She has been in physical therapy, taken oral analgesics, and
had epidural injections with minimal relief. What is the best next step?

A. Lateral interbody fusion


B. Laminectomy
C. Posterior spinal fusion with multilevel transforaminal lumbar interbody fusion
D. Microdiskectomy

R: B
The patient has lumbar stenosis of L2-3 and L3-4. She has no spondylolisthesis or instability.
For her condition, spinal fusion plays a minimal role. She has no evidence of instability, and
her condition can be addressed through laminectomy only. No role exists for
microdiskectomy, because her disease results from a combination of ligamentum flavum
hypertrophy and facet hypertrophy.

Question 50 of 100
A 54-year-old man is diagnosed with a T6 chordoma. Which procedure can provide
the least chance of recurrence?

A. En bloc spondylectomy
B. Denosumab injection
C. Preoperative embolization
D. Postoperative radiation
Adult Spine Scored and Recorded Self-Assessment Examination 2020

R: A
When feasible, en bloc resection is associated with the least recurrence in the surgical
management of chordomas. Denosumab has been used for the treatment of giant cell
tumors, along with surgical resection. Preoperative embolization has not been associated
with the prevention of recurrence. Postoperative radiation can supplement en bloc resection
but is not a stand-alone modality that can prevent resections.

Question 51 of 100
What clinical scenario is most consistent with the MR image of the L4-L5 disk level
shown in Figure 1?

A. Left L4 nerve root radiculopathy


B. Right L5 nerve root radiculopathy
C. Bowel and bladder dysfunction
D. Symptomatic pseudomeningocele

R: A
The MRI scan reveals a foraminal disk herniation originating from the L4-L5 disk space that
has migrated into the foramen compressing the left L4 nerve root. There is no evidence of
compression of the right L5 nerve root. Bowel and bladder dysfunction are not associated
with L4-mediated nerve function. There is no evidence of pseudomeningocele.

Question 52 of 100
Figure 1 is the radiograph of a 51-year-old with back pain and right leg pain. The
patient has a positive straight leg raise, full strength in the bilateral lower extremity,
as well as intact sensation. What is the most common cause of the radicular leg pain?
Adult Spine Scored and Recorded Self-Assessment Examination 2020

A. Foraminal stenosis
B. Hypertrophic ligamentum flavum
C. Lateral recess stenosis
D. Stretching of the nerve root

R: A
The radiograph reveals an L5-S1 spondylolisthesis secondary to L5 spondylolysis. Patients
with isthmic spondylolisthesis have fibrous tissue at the pars interarticularis, which
contributes to bilateral L5-S1 foraminal stenosis. This typically results in L5 radiculopathy,
which is the exiting nerve root at L5-S1. Lateral recess stenosis and hypertrophic
ligamentum flavum are typically seen in degenerative spondylolisthesis.

Question 53 of 100
After direct lateral (transpsoas) interbody fusion surgery at L3-4, a patient reports
numbness in the scrotum, and ipsilateral anterior thigh pain develops. What is the
most likely cause?

A. Injury to the genitofemoral nerve


B. Injury to the ilioinguinal nerve
C. Injury to the femoral nerve
D. Prolonged lateral decubitus positioning

R: A
The genitofemoral nerve is at risk at almost any level in the lateral transpsoas approach.
The nerve provides sensory innervation to the anterior thigh and scrotum/labia. The
ilioinguinal nerve provides sensory innervation to the mons pubis or labia in women and the
upper scrotum in men. The femoral nerve is responsible for sensation to the anterior and
Adult Spine Scored and Recorded Self-Assessment Examination 2020

medial aspects of the thigh, leg, and medial foot. It also provides innervation to knee
extensor muscles. Prolonged decubitus positioning, especially with jackknife
hyperextension, can cause stretching of the femoral nerve and transient weakness of the
ipsilateral quadriceps.

Question 54 of 100
Figures 1 and 2 are the radiograph and MRI scans, respectively, of a 35-year-old
woman who is injured in a small plane crash. Despite being seat-belted, she sustains
a severe distal tibial fracture. She is conscious and complains of back and leg pain.
She is neurologically intact. What is the most appropriate next step in management?

A. Multi-level posterior spinal fusion


B. Anterior spine fusion
C. Abdominal CT scan
D. Bed rest
R: C
Flexion-distraction injuries of the spine are frequently associated with concomitant intra-
abdominal injuries including hollow viscus injuries, mesenteric tears, and liver and spleen
injuries. This is especially evident in seat-belt related motor vehicle collisions. Often patients
with seat-belt injuries will have abdominal bruising or contusions that should be looked for
on initial evaluation. General surgical or trauma team evaluation includes abdominal
evaluation typically with CT evaluation of the abdomen or peritoneal lavage. Treatment of
the spinal injury especially in a neurologically intact patient. should be delayed until proper
evaluation for abdominal injuries with this fracture pattern.

Question 55 of 100
Figures 1 through 3 are the MR images and CT scan of a 65-year-old man with a
history of diabetes mellitus, hypertension, and smoking. He has a 6-week history of
increasing midback pain, lower extremity pain, and weakness. What is the most likely
diagnosis, and how should this diagnosis be confirmed?
Adult Spine Scored and Recorded Self-Assessment Examination 2020

A. Thoracic tumor; CT-guided biopsy


B. Thoracic tumor; 3-phase bone scan
C. Thoracic diskitis and osteomyelitis; CT-guided biopsy
D. Thoracic diskitis and osteomyelitis; tagged white blood cell scan

R: A
The sagittal T2-weighted and axial T2-weighted images show a lesion within the T8 vertebral
body that involves the posterior elements. There is an associated epidural component that
results in compression of the spinal cord. The sagittal reconstructed CT image shows a lytic
lesion within the T8 vertebral body. This pattern of vertebral body involvement with
preservation of the adjacent disks and endplates in a 65-year-old patient is most compatible
with a diagnosis of a tumor. The most likely tumor is a metastatic lesion. A CT-guided biopsy
will confirm this diagnosis. Although thoracic tuberculosis does not typically cross the disk
space, the lack of an anterior soft-tissue component decreases the likelihood of this
diagnosis.

Question 56 of 100
Figures 1 through 4 are the MRI scans and radiograph of a 48-year-old woman who
has chronic neck pain and severe lower back pain. The pain is constant and seems
to worsen at night. Other subjective complaints include: sleep disturbance, difficulty
maintaining concentration, chronic fatigue, migraine headaches, morning stiffness,
forgetfulness, and difficulty with balance. She also describes frequent episodes of
bilateral lower extremity burning-like pain. On examination, she has normal motor
and sensory function in upper and lower extremities. She does, however, have
multiple sites of palpable tenderness scattered throughout her body. She has tried
massage therapy and chiropractic management in the past with marginal results.
Imaging is provided. What is the most appropriate treatment?
Adult Spine Scored and Recorded Self-Assessment Examination 2020

A. Lumbar interbody arthrodesis


B. Lumbar and cervical epidural injections
C. Prescribing opiates and anxiolytics
D. Pharmacologic neuromodulation

R: D
The patient has a diagnosis of fibromyalgia, which the American College of Rheumatology
defines as chronic widespread pain with at least 11 of 18 possible tender points. Etiology is
multifactorial, but there is a genetic predisposition. Some associated factors include: history
of widespread pain and hyperalgesia, sleep disorder, inactivity, functional disability,
concomitant anxiety or mood disorder, and fear avoidance behavior. The most effective
treatment for fibromyalgia is multimodal; however, pharmacologic neuromodulation (use of
antiepileptic drugs, tricyclic antidepressants, selective serotonin-reuptake inhibitors, and
serotonin norepinephrine reuptake inhibitors) has been found to be moderately successful
in treating widespread pain and hyperalgesia. These medications are aimed at altering the
neurochemistry of the central nervous system and diminishing the perception of pain. Other
treatments include physical activity to address inactivity and functional disability, sleep
quality improvement, interventions treating anxiety and mood disorders, and interventions
targeting fear-avoidance behavior. Surgery, specifically fusion procedures, has not been
found to be effective in such patients with discogenic changes only; without instability or
stenosis. There is no high-level study showing efficacy of either lumbar or cervical epidural
injection in patients with fibromyalgia who complain of neck or back pain without
radiculopathy. The use of opiates and anxiolytics should be closely monitored because this
patient population is especially at high risk for abuse and dependence.

Question 57 of 100
A 78-year-old man is seen in the emergency room 3 hours after a fall from a standing
position. The patient sustained a mild scalp laceration and the injury shown in Figure
1. He reports severe neck pain and is unable to move his hands and legs. Examination
reveals absent motor function in the wrist flexors, triceps, and fingers. He cannot
move his lower extremities during motor testing. The patient has some sensation in
the lower extremities. Bulbocavernosus reflex is absent. Based on examination
findings and the imaging findings, what is the most definitive treatment option?
Adult Spine Scored and Recorded Self-Assessment Examination 2020

A. Halo application and cervical traction for 6 weeks, followed by 8 weeks of immobilization
in a halo-thoracic vest
B. Open reduction, decompression, and fusion with anterior-posterior stabilization
C. Open reduction, anterior decompression, and fusion
D. Uninstrumented posterior fusion spanning the injured segment

R: B
The patient has a hyperostotic condition of the cervical spine, most likely ankylosing
spondylitis. Because of a rigid and osteoporotic spine, relatively minor falls can result in
unstable spinal injuries with significant instability and a high risk for neurologic sequelae.
The patient has an unstable injury at C6 with an incomplete spinal cord injury, necessitating
urgent decompression and stabilization. Studies have shown that, in patients with
ankylosing spondylitis, stand-alone anterior stabilization results in a high failure rate. Halo-
thoracic vests carry a high risk of septic and pulmonary issues, especially in the elderly.
Uninstrumented fusion will provide insufficient stability in such patients.

Question 58 of 100
A 45-year-old woman recently had a cervical epidural injection to treat her cervical
radicular symptoms. She is at the emergency department 3 days following the
injection with worsening neck pain, headaches, and malaise. On examination, she is
neurologically intact. MRI of the cervical spine shows an epidural abscess (Figure 1),
and blood cultures are negative. If nonsurgical treatment is pursued, what factor is
most predictive of success of nonoperative management?
Adult Spine Scored and Recorded Self-Assessment Examination 2020

A. Diagnosis of diabetes mellitus


B. Active malignancy
C. Concurrent vertebral body fracture
D. Dorsal location of the epidural abscess

R: D
Cervical epidural injections have been associated with cervical epidural abscesses and
spinal cord injury. This patient’s history of recent cervical epidural injection should raise a
suspicion of epidural abscess and prompt additional imaging. In a series of 367 patients with
epidural abscesses, Shah and associates identified 99 patients who failed nonsurgical
management. Factors predictive of failure of nonsurgical management included: motor and
sensory deficits, compression/pathologic fracture, active malignancy, and diabetes mellitus.
Dorsal location of the epidural abscess was predictive that nonsurgical management can
succeed, as opposed to ventral location of the abscess.

Question 59 of 100
An obtunded 80-year-old man was found alone in his apartment after an apparent fall.
A CT scan performed in the emergency department shows that he has an extensile
injury of an ankylosed cervical spine. The fracture extends across the ossified C5-C6
disk space and into the lamina of C5. There is 1.5 cm of widening between the C5 and
C6 vertebrae anteriorly. The patient's family asks you about the long-term impact of
the fracture on his functional capacity and survival. You advise them that patients
with fractures of the cervical spine with ankylosing conditions have

A. an unknown prognosis until they can participate in an examination.


B. higher rates of neurologic deficit and mortality than other same-age people.
C. higher rates of neurologic deficit than other same-age people, but comparable mortality
rates.
Adult Spine Scored and Recorded Self-Assessment Examination 2020

D. similar rates of neurologic deficit and mortality as other same-age people.

R: B
Several studies have found that rates of neurologic deficit and mortality are higher for
patients with ankylosing spondylitis and a spinal fracture than for age-matched controls. The
2011 work of Schoenfeld and associates, which directly compared patients with cervical
fractures in ankylosed spines to age- and sex-matched controls who also had cervical
fractures but no ankylosing condition, demonstrated that those with ankylosing spondylitis
were at elevated risk for mortality for up to 2 years after sustaining a fracture. In a study by
Westerveld and associates, the rate of neurologic deficit among patients with ankylosing
spondylitis and a spinal fracture was 57.1% compared to 12.6% among controls.

Question 60 of 100
Figure 1 is the MRI scan of a patient with left leg pain. Which nerve root is most likely
affected?

A. L2
B. L3
C. L4
D. L5

R: B
Central/posterior lateral disk herniations affect the traversing nerve root. Comparatively,
foraminal disks or extra foraminal (far lateral) disks affect the exiting nerve root. Here, a far
lateral disk is seen on the right of the image (patient’s left side) at L3-L4. This would affect
the L3 nerve root once it has exited the neural foramen. Patients would experience pain on
the anterior thigh and potentially weakness in the quadriceps.

Question 61 of 100
Figure 1 is the MRI from a 67-year-old man with severe neck pain 1 week following
dental extraction. He has a history of poorly controlled type 2 diabetes mellitus. On
examination, he is found to have grade 4 of 5 strength in the bilateral lower extremities.
Adult Spine Scored and Recorded Self-Assessment Examination 2020

He is febrile and has an elevated erythrocyte sedimentation rate and an elevated C-


reactive protein level. His MRI reveals an epidural abscess. What is the best next step?

A. Intravenous antibiotics
B. Observation and intravenous antibiotics
C. Surgical decompression
D. Interventional radiology drainage

R: C
The patient has an epidural abscess following a dental procedure. The epidural abscess
spans from C2 to the upper thoracic spine. He has severe neck pain, neurologic changes,
and elevated laboratory markers. Sang and associates have demonstrated that, in patients
older than 65 years with a methicillin-resistant Staphylococcus aureus infection, a history of
diabetes, and neurologic deficits, nonsurgical management has a 99% chance of failure.
Prompt surgical decompression to evacuate the abscess followed by antibiotic treatment is
the best method of treatment for this patient.

Question 62 of 100
A 60-year-old woman is at the emergency department with a one-week history of
thoracic back pain along with fevers >103°F. She notes that her legs are becoming
slightly numb over the last 24 hours, but she is able to walk normally and she has a
normal lower extremity motor examination. Her medical history is significant for
diabetes mellitus. Her WBC is 13x109 cells/liter. She would like to avoid surgery. Her
T1-weighted post gadolinium MRI scans are shown in Figures 1 and 2. What should
you advise her regarding her risks?
Adult Spine Scored and Recorded Self-Assessment Examination 2020

A. She is at risk for developing a motor deficit.


B. She is at low risk for developing a motor deficit.
C. She is at low risk for developing a motor deficit if steroids are started immediately.
D. Her WBC count and history of diabetes are not relevant.

R: A
In a retrospective study of patients with epidural abscess from two academic medical centers,
a predictive algorithm was developed to help identify which patients will develop a motor
deficit. Multivariate analysis allowed points to be assigned to each risk factor. A sensory
deficit was associated with 10 points; urinary retention/incontinence, 8 points; fecal
incontinence/retention, 5 points; abscess above the conus medullaris, 4 points; diabetes, 2
points; WBC count >12x109 cells/liter, 2 points, and the presence of multiple epidural
abscesses, 4 points. Smoking was not found to be predictive of a motor deficit. A dorsally
based abscess was found to be protective of having a deficit, but a ventral or circumferential
abscess was not. The use of steroids in the setting of infection is not recommended.

Question 63 of 100
Figures 1 through 3 show the MRI scans of a 56-year-old woman with progressively
worsening low back and bilateral lower extremity pain. Based on these images, what
muscle or muscle group would be expected to be weak on physical examination?
Adult Spine Scored and Recorded Self-Assessment Examination 2020

A. Quadriceps
B. Hamstrings
C. Extensor hallucis longus
D. Gastrocnemius

R: C
Whereas subjective complaints of leg pain are common among patients seeking surgical
treatment for spondylolisthesis, documented neurologic deficit or radiculopathy is seen less
frequently. Subjective decreases to light touch over the dorsum of the foot and mild
weakness of the extensor hallucis longus are the most common neurologic abnormalities,
correlating with L5 nerve root irritation as seen with L5-S1 spondylolisthesis. Many patients
with spondylolisthesis report hamstring tightness; however, these structures are not usually
weak. Quadriceps and tibialis anterior weakness is seen with L4 nerve root irritation. The
gastrocnemius is generally weak in S1 nerve root syndromes.

Question 64 of 100
In medical malpractice cases against spine surgeons, what factor is associated with
a judgement for the plaintiff?

A. Orthopaedic spine surgeon as the defendant


B. Neurosurgeon as the defendant
C. Male patient
D. Case of diagnostic delay

R: D
Medical litigation is common in spine surgery. In a study evaluating “spine surgery” related
legal cases from 1988 to 2015, 234 cases met the inclusion criteria. Diagnostic delay cases
were significantly associated with plaintiff verdict or settlement. Therapeutic delay cases
Adult Spine Scored and Recorded Self-Assessment Examination 2020

were also associated with plaintiff verdict or settlement. Catastrophic complications resulted
in larger payouts (6.1 million) as compared with noncatastrophic complications (2.9 million).
There is no association between specialty (neurosurgery or orthopaedic spine surgery),
patient age/sex, and case outcome or award.
Question 65 of 100
During the approach to the lumbar spine for an L4-L5 anterior lumbar interbody fusion,
which structure generally is found overlying the anterior surface of the L4 vertebra?

A. Aorta
B. Right common iliac artery
C. Left common iliac vein
D. Right ureter

R: A
During an anterior approach to the L4-L5 disk space for anterior lumbar interbody fusion,
meticulous exposure is paramount to allow for safe preparation of the disk space and
subsequent arthrodesis. Although all of these structures can come into play during the
exposure, the aorta lies anterior to the L4 vertebral body and bifurcates at this level. The
vena cava bifurcates just distal to this. The ureters lie to both sides of the anterior spine. The
right common iliac artery and the left common iliac vein originate after the bifurcation of the
great vessels and lie caudal to the L4 vertebra.

Question 66 of 100
What outcome measuring tool focuses on reliability; precision; and versatility, while
mitigating administrative burden?

A. Oswestry Disability Index (ODI)


B. Neck Disability Index (NDI)
C. Patient-Reported Outcomes Measurement Information System (PROMIS)
D. 36-item Short-Form Physical Functioning scale (SF-36)

R: C
Several clinical outcome measuring tools have been used in orthopaedics to assess health
status in clinical care, research, and cost-effective analysis. PROMIS was designed to focus
on psychometric characteristics, which would render it precise, reliable, and versatile.
PROMIS also uses computerized adaptive testing (CAT) in contrast with conventional
outcome measures (SF-36, ODI, or NDI). With CAT, an algorithm customizes item delivery
based on responses to previous items. This enables precision with fewer questions and
mitigates examinee fatigue or loss of focus. PROMIS also uses an easily understandable T-
score (normalized to general population) as an output. A score of 50 is set as the mean, and
the standard deviation is set at 10 points. PROMIS has been compared with conventional
measures (general health and disease-specific patient related outcome measures) and has
been found to improve coverage of relevant health domain, increase reliability, and reduce
Adult Spine Scored and Recorded Self-Assessment Examination 2020

respondent and administrative burden. PROMIS has been extensively studied in the
following orthopaedic disorders: foot and ankle, upper extremity, and spine.

Question 67 of 100
An 80-year-old man who was involved in a fall from ground height is evaluated in the
emergency department for head lacerations and mild neck pain. Examination reveals
only mild tenderness of the posterior neck region with some limitation of motion.
Neurologic examination is normal. Radiographs of the cervical spine are shown in
Figures 1 and 2. What is the next most appropriate step in management for this patient?

A. Admission for 24-hour observation for possible intracranial hematoma


B. CT of the cervical spine
C. Repeat radiographs of the cervical spine, including flexion and extension views
D. Hard cervical collar for 6 weeks

R: B
The patient has radiographic findings compatible with diffuse idiopathic skeletal hyperostosis
(DISH) of the cervical spine. Characteristics of DISH include flowing, non-marginal
osteophytes at four or more levels. Patients with DISH develop a significant loss of flexibility
of the spine. The spine acts more as a long bone with minimal force needed to create
unstable fractures. Any minor trauma in patients with DISH should be worked up
aggressively to rule out occult fracture. In this patient, radiographs fail to clearly rule out a
fracture; therefore, CT of the cervical spine is indicated. Without a suspicion of history of a
head injury, admission specifically for a possible intracranial hematoma is not warranted.
The more concerning injury in a patient with DISH is occult neck fracture. Treatment with a
soft or hard collar is not advised until a fracture is ruled out. Repeat radiographs are unlikely
to show any occult fractures, and flexion and extension views would not be advised in a
patient with a suspected vertebral fracture.
Adult Spine Scored and Recorded Self-Assessment Examination 2020

Question 68 of 100
An 83-year-old woman has leg pain with ambulation. She has tried physical therapy,
oral analgesics, and injections, with minimal relief. The symptoms have been present
for 1 year. Radiographs reveal an L4-5 spondylolisthesis and greater than 4 mm of
motion on flexion-extension. MRI shows moderate to severe central and lateral recess
stenosis. The patient should be informed that at her age, surgical intervention

A. can provide considerable benefit compared with nonsurgical management.


B. should be avoided because of a higher rate of complications.
C. has no difference in results from nonsurgical management.
D. can lead to increased mortality.

R: A
The incidence of surgery is increased in patients 80 years of age and older. Patients aged
80 years and older enrolled in the Spine Patient Outcomes Research Trial and undergoing
surgery for lumbar stenosis and spondylolisthesis were compared with patients younger
than 80. In the older age group, surgical treatment was associated with statistically
significant clinical improvement compared with nonsurgical management. No statistically
significant increase was observed in complications or mortality compared with younger
patients.

Question 69 of 100

Figure 1 shows the standing lateral radiograph of a 62-year-old woman who reports
lower back pain and the inability to stand upright. What permanent anatomic pelvic
parameter should be measured and considered when determining the amount of
lumbar lordosis correction that will be necessary to obtain sagittal balance?

A. Pelvic tilt
B. Pelvic incidence
Adult Spine Scored and Recorded Self-Assessment Examination 2020

C. Sacral slope
D. Acetabular version

R: B
Pelvic incidence (PI) is the anatomic angle between the sacral end plate and a line
connecting the center of the femoral heads. Increased pelvic incidence has been found to
correlate with the incidence and severity of spondylolisthesis. Patients with increased PI
require increased lumbar lordosis to restore sagittal balance. Pelvic tilt (PT) and sacral slope
(SS) have also been found to correlate with lumbar lordosis; however, both PT and SS can
change depending on pelvic rotation. PI is the only permanent pelvic parameter that is
unaffected by pelvic rotation. Acetabular version has not been found to be associated with
lumbar lordosis.
Question 70 of 100
A 56-year-old man has upper thoracic pain after undergoing stereotactic radiosurgery
for a blastic metastatic lesion in the vertebral body of T5. He has normal alignment
without collapse, but the tumor involves the entire vertebral body. Which factor
increases this patient's risk for a pathologic fracture?

A. Location of the lesion at T5


B. History of stereotactic radiation
C. Blastic nature of the tumor
D. Tumor involving all of the vertebral body

R: B
Criteria for spinal instability have been outlined by the Spine Oncology Study Group, which
developed the Spinal Instability Neoplastic Score (SINS) criteria. Factors associated with
lower risk for instability/fracture are location outside of a junctional level (the SINS criteria
use C7-T2 as the junctional level), blastic metastases, and no evidence of vertebral collapse
(even with more than 50% involvement). Radiation has been associated with risk for
pathologic fracture, and stereotactic radiation has been associated with risk for a spinal
fracture.

Question 71 of 100
Current U.S. Food and Drug Administration (FDA)-approved indications for cervical
disk replacement include

A. 1-2 level cervical spondylosis with radiculopathy.


B. axial neck pain.
C. cervical kyphosis correction.
D. patients <70 years.

R: A
Adult Spine Scored and Recorded Self-Assessment Examination 2020

Cervical disk replacement is indicated for 1-2 levels depending on the chosen implant.
Studies have examined its use in patients <60 years for symptomatic cervical radiculopathy
and/or myelopathy.

Question 72 of 100
A 60-year-old woman has progressive neck pain, upper extremity pain, and
paresthesias. A lateral cervical spine radiograph and an MRI scan are shown in
Figures 1 and 2. What is the most likely underlying diagnosis?

A. Osteomyelitis
B. Ankylosing spondylitis
C. Age-related degenerative changes
D. Rheumatoid arthritis

R: D
The radiograph and sagittal T2-weighted MRI scan show multilevel degenerative changes
and subaxial subluxations with anterolisthesis at C3-C4 and C4-C5 and retrolisthesis at C5-
C6. In addition, there is evidence of midcervical kyphosis. Such findings are often seen in
patients with rheumatoid arthritis. Patients with osteomyelitis typically show increased signal
intensity in the disks and vertebral bodies. Patients with ankylosing spondylitis typically show
ankylosis of the disks and vertebral bodies. Age-related degenerative changes typically
manifest as degenerative disk disease with occasional single-level spondylolisthesis, but not
typically multilevel spondylolisthesis, as seen in this patient. The spinous processes are
intact; these changes do not appear to be postoperative.

Question 73 of 100
A 73-year-old woman has back and leg pain. Imaging reveals a lumbar degenerative
scoliosis. Nonsurgical management, consisting of physical therapy, medications, and
injections, has failed. During the surgical planning, dual-energy x-ray absorptiometry
is performed, and her T-score returns as -2.6. Intraoperative options to help reduce
the risk of instrumentation failure include
Adult Spine Scored and Recorded Self-Assessment Examination 2020

A. teriparatide injection.
B. multilevel interbody fusion.
C. augmentation of pedicle screws with polymethylmethacrylate (PMMA).
D. iliac crest bone graft.

R: C
Instrumentation of the osteoporotic spine is becoming more common as the population ages.
Some intraoperative options to reduce pedicle screw failure rates include augmenting the
pedicle screw with PMMA, using a fenestrated screw designed for injection of the PMMA
through the screw, and using hydroxyapatite coated screws. Teriparatide is a parathyroid
hormone analogue used as a second-line treatment for osteoporosis. Preoperative
administration potentially can increase bone quality. Postoperative administration of
teriparatide has been shown to increase lumbar fusion rates. In the setting of osteoporosis,
multilevel interbody fusion can increase the risk of implant subsidence. Although iliac crest
bone graft is the gold standard graft used to obtain fusion, it does not have immediate impact
on the rate of implant failure in osteoporosis.

Question 74 of 100
An otherwise healthy 47-year-old woman has numbness and weakness in her bilateral
lower legs. Her physical examination reveals an ataxic gait and +3/4 patella and
Achilles reflexes. Her upper extremity neurological examination is normal. Her MRI
scans are shown in Figures 1 through 3. What additional test would best help you
plan her treatment?

A. Flexion / extension radiographs of the thoracic spine


B. CT scan of the thoracic spine
C. MRI scan of the cervical spine
D. Pulmonary function test

R: B
Adult Spine Scored and Recorded Self-Assessment Examination 2020

The patient has thoracic disk herniation. This is causing significant compression of her spinal
cord. Her symptoms and physical examination are consistent with myelopathy. Given the
patient’s symptoms, surgical treatment is most appropriate. A significant number of thoracic
disk herniations will calcify, which can alter the surgical approach. A CT scan (or CT
myelogram) is the best way to detect calcification. Flexion extension images, while helpful,
may not detect the calcified disk. Instability in the thoracic spine is uncommon. Given the
patients hyperreflexia, lumbar pathology contributing to the patient’s complaints is unlikely.
Pulmonary function tests may be considered for patients requiring a transthoracic approach,
but they are unlikely to be abnormal in an otherwise healthy patient.

Question 75 of 100
Figures 1 through 3 show the CT and radiographs of a 45-year-old woman who has
low back pain and severe left leg pain in the L5 distribution that is not responsive to
nonoperative management (physical therapy, epidural injections, pain medications)
for 9 months. What is an appropriate treatment option for this patient?
Adult Spine Scored and Recorded Self-Assessment Examination 2020

A. Percutaneous cyst aspiration


B. Complete left facetectomy and cyst removal without instrumentation and fusion
C. Posterior lumbar decompression and fusion with instrumentation
D. Indirect lumbar decompression with lateral lumbar interbody fusion

R: C
The figures show a grade 1 spondylolisthesis at L4-L5 along with a left sided facet cyst in a
patient with ongoing symptoms despite nonoperative management. A lumbar
decompression with a fusion would be an appropriate treatment option in this patient. There
is no evidence in the literature for a lateral lumbar interbody fusion without directly
decompressing the spinal canal for treatment of spondylolisthesis
in the presence of a large facet cyst. Such procedures have been shown to be successful
for treatment in the presence of spinal stenosis from ligament hypertrophy, disk bulge,
foraminal stenosis. However, there is no evidence for their effectiveness in the presence of
a large facet cyst. Similarly facet cyst aspiration has no evidence for treatment of this
pathology. A facetectomy and cyst removal may adequately decompress the L4-L5 level.
However, this is likely to create further instability in the presence of a spondylolisthesis.

Question 76 of 100
When posteriorly instrumenting the lumbar spine in the scoliosis shown in the
radiograph in Figure 1, a compressive force across the convexity of the curve will
result in

A. kyphosis.
B. lordosis.
Adult Spine Scored and Recorded Self-Assessment Examination 2020

C. cantilever.
D. rotation.

R: B
When instrumenting the spine posteriorly, distraction forces posterior to the axis of rotation
result in kyphosis and compression forces result in lordosis. Thus, when correcting a lumbar
scoliosis deformity posteriorly, the convex rod is placed first with a compressive force to
obtain scoliosis correction, as well as improved lumbar lordosis. A distraction force typically
follows this in the concavity of the curvature. Similarly, distraction forces anterior to the axis
of rotation with anterior spinal instrumentation will result in lordosis, and compression forces
will result in kyphosis. Rotation is obtained by exerting a force in the axial plane. Cantilever
forces occur when a rod is rigidly attached to one end of a spinal deformity and a load is
used to create a moment at the point of attachment to the support, i.e. used in kyphosis
correction.

Question 77 of 100
Figures 1 and 2 are MR images of a 34-year-old man who is referred to your office by
his primary care physician after failing 4 months of nonsurgical treatment that
included epidural steroids for severe right arm pain occurring in a C6 distribution. He
also has associated paresthesias in this region. The patient is weak in elbow flexion
and wrist extension. What are his likely outcomes if he is treated with a posterior
foraminotomy instead of anterior cervical diskectomy and fusion (ACDF)?

A. Similar incidence of postsurgical neck pain with higher risk for radiculopathy recurrence
at the same level
B. Higher incidence of postsurgical neck pain and radiculopathy recurrence at the same
level
C. Higher incidence of postsurgical neck pain and adjacent-level radiculopathy
D. Lower incidence of adjacent segment degeneration and postsurgical neck pain
Adult Spine Scored and Recorded Self-Assessment Examination 2020

R: B
This patient has a right-sided C5-C6 disk herniation causing C6 radicular symptoms in the
right upper extremity. Studies have shown that both ACDF and posterior foraminotomy
confer similar results in terms of pain relief and functional outcome. Patients treated with
posterior foraminotomy are at higher risk for neck pain and recurrence of radiculopathy at
the same level. Those who receive ACDF are at higher risk for occurrence of radiculopathy
at an adjacent level.

Question 78 of 100
Interspinous devices work by distracting the posterior elements and widening the
spinal canal via blockage of the spinous process. It can be performed with or without
a decompression. The use of interspinous devices increases

A. Oswestry Disability Index (ODI) score.


B. Visual Analog Scale (VAS) score.
C. Facet loading.
D. Focal kyphosis.

R: D
Interspinous devices are utilized to mitigate the symptoms of neurogenic claudication
secondary to lumbar spinal stenosis with forced forward flexion. Interspinous devices can
be classified as a distracting device or a stabilizing device. The inhibition of extension with
a blocking device widens the central canal and foraminal height and decreases the load on
the facet joints. Various types of interspinous devices have been shown to decrease the ODI
and VAS scores.

Question 79 of 100
Figures 1 and 2 are the radiographs of a patient who was involved in a motor vehicle
collision. He was wearing his seat belt and is now complaining of midthoracic back
pain. Radiographs in the emergency department do not reveal a fracture. What is the
most appropriate next step?
Adult Spine Scored and Recorded Self-Assessment Examination 2020

A. Nonsteroidal medication and follow up as needed


B. Repeat radiographs in 1 week
C. MR imaging of the thoracic spine
D. Flexion-extension radiographs

R: C
Ankylosing spinal disorders, including ankylosing spondylitis and diffuse idiopathic skeletal
hyperostosis, are conditions that make the spine rigid and at risk for 3-column unstable
fractures. Spinal fractures in these patients pose high risk for complications and death and
patients should be counseled and observed closely. Mortality strongly correlates with older
age and increased number of comorbidities.
These spine fractures often are not seen at the time of initial evaluation, and a delay in
diagnosis can occur in up to 19% of cases. This is particularly common in the setting of non-
or minimally displaced fractures following minor injuries. A delayed diagnosis can lead to
displacement of a previously nondisplaced fracture that can incur a high neurologic injury
risk. Advanced imaging with a CT scan or MRI should be obtained for patients with
ankylosing spinal disorders even when minor injuries occur. Although bracing and
observation can be used, posterior multilevel spinal instrumentation is typically required to
obtain adequate spinal stabilization.
The radiographs show an osteopenic ankylosed thoracic spine; the anteroposterior
radiograph clearly shows fusion of the sacroiliac joints. Recognition of these radiographic
findings is important when evaluating patients after an injury.

Question 80 of 100
With regards to cervical surgery, the perioperative risk of venous thromboembolic
(VTE) disease is increased by

A. posterior surgery.
B. female gender.
C. anterior surgery.
D. Hgb >10.

R: A
Oglesby and associates evaluated the incidence of VTE after 273,000 cervical procedures
using a National Inpatient Sample Database (from 2002 to 2009). Risk factors for deep
venous thrombosis (DVT) and pulmonary embolism were stratified. The overall rate of VTE
was 5 per 1,000 procedures. Specific increased risk factors include: posterior cervical fusion
with an incidence of 13.4 per 1,000 patients (odds ratio 2.3), male gender (odds ratio 1.8),
fluid and electrolyte imbalance (odds ratio 2.2), postoperative anemia (odds ratio 4.8), and
pulmonary vascular pathology (odds ratio 3.7).
Adult Spine Scored and Recorded Self-Assessment Examination 2020

Question 81 of 100
A 56-year-old man has had a 2-year history of slowly progressive neck pain and
bilateral arm aching. Over the past year, he has noticed intermittent, diffuse
numbness in both hands, with decreased grip strength and mild hand clumsiness. He
denies any problems with balance. Examination shows a wide-based gait, intrinsic
wasting, and a positive Hoffman's sign bilaterally. An MRI scan of the cervical spine
is shown in Figure 1. What is the most appropriate treatment?

A. Epidural injections
B. Anterior diskectomy and fusion at C4-C5 and C5-C6
C. Multilevel laminectomy and fusion
D. Multilevel posterior foraminotomies

R: C
The patient has classic symptoms of myelopathy with upper motor neuron signs on
examination. His symptoms have been present for years, and are getting worse. The
cervical spine MRI scan shows spinal stenosis with multilevel spondylosis causing spinal
cord compression at multiple levels. With the longstanding duration of the patient's signs
and symptoms, combined with involvement of multiple levels in the cervical spine, posterior
multilevel laminectomy and fusion is the best treatment option. Two-level anterior
diskectomy and fusion would address the two areas of most severe narrowing, but it would
fail to decompress the other stenotic areas which also require decompression. Posterior
cervical foraminotomies would only address radicular symptoms, which are not present in
this patient, and would not succeed in decompression of the spinal cord. Cervical epidural
injections are not indicated for myelopathy symptoms, and may in fact place this patient at
risk for neurologic deterioration.

Question 82 of 100
A 24-year-old man is involved in a motor vehicle collision. A CT scan of the cervical
spine shows the injury in Figures 1 and 2. He had a normal motor and sensory
examination. What is the most appropriate treatment option for this injury?
Adult Spine Scored and Recorded Self-Assessment Examination 2020

A. Immobilization in a cervical collar


B. Halo traction
C. Anterior cervical diskectomy and fusion or posterior cervical instrumentation and fusion
C4-C5
D. Anterior cervical diskectomy and fusion or posterior cervical instrumentation and fusion
C3-C4, C4-C5

R: D
The figures show a unilateral floating mass fracture of C4 with horizontalization of the C4
facet on the left side and <25% anterior listhesis at C4-C5. These injuries are considered 2
level injuries; therefore, the injury in this patient is a C3-C4 and C4-C5 injury. Nonoperative
treatment has been found to be unsuccessful in managing these injuries and lead to
subluxation over time. Surgical fixation of the two involved levels, either anteriorly or
posteriorly is acceptable. Surgical treatment of only one of the levels may leave the instability
at the second level unaddressed.

Question 83 of 100
What is the chief mechanism of action of parathyroid hormone (PTH) in the treatment
of patients with osteoporosis?

A. Reduces resorptive activity of osteoclasts


B. Inhibits receptor activator of nuclear factor kappa-B ligand
C. Stimulates osteoblastic bone formation
D. Acts as a selective estrogen modulator

R: C
Recombinant human PTH benefits patients with osteoporosis by stimulating osteoblastic
bone formation and reducing osteoblastic apoptosis. Treatment reduces vertebral fractures
by 65%. PTH analogs act similarly and reduce vertebral fractures by 47%. Bisphosphonates
reduce the resorptive activity of osteoclasts and cause a dissociation of bone formation and
resorption that favors bone formation and reduce vertebral fractures by 50% to 70%.
Selective estrogen receptor modulators inhibit bone resorption and reduce vertebral
Adult Spine Scored and Recorded Self-Assessment Examination 2020

fractures by 35%. Humanized monoclonal antibodies inhibit osteoclast formation and reduce
vertebral fractures by 68%.

Question 84 of 100
A 20-year-old woman involved in a motor vehicle collision sustains a C4 burst fracture
and an incomplete spinal cord injury. The patient undergoes urgent surgical
decompression and stabilization. She is admitted to the intensive care unit with an
arterial line place and mean arterial pressure (MAP) elevated to >85 mmHg. What is
the underlying premise for elevating the MAPs following spinal cord injury?

A. Regenerate neural stem cells


B. Decrease spinal cord ischemia and increase perfusion toinjured neural tissue
C. Prevent neurogenic shock
D. Increase blood supply to lower extremity musculature

R: B
Maintaining MAPs 85 mmHg to 90 mmHg has been advocated by the American Association
of Neurological Surgeons/Congress of Neurological Surgeons for up to 7 days to increase
spinal cord perfusion. The thought is that by increasing MAPs, spinal cord ischemia can be
avoided. An intensive care unit stay is often needed to monitor the MAPs and vasopressors
may be needed. From the vasopressors used (dopamine, norepinephrine, phenylephrine),
dopamine has led to most complications. There has been a case report of increased MAP
use in the setting of incomplete spinal cord injury leading to posterior reversible
encephalopathy syndrome. Randomized trials to determine which MAP goal is ideal are
undergoing.

Question 85 of 100
A 78-year-old woman has a history of chronic low back pain. She denies any extremity
problems. Her pain is worse in the morning, and gets better, although it does not go
away, as the day goes on. An MRI scan of the lumbar spine is shown in Figure 1. She
denies any acute worsening of her symptoms, although in general, her symptoms are
slowly worsening. She takes nonsteroidal anti-inflammatory drugs as needed for her
pain, but otherwise takes no other medications. What is the next most appropriate
step in management?
Adult Spine Scored and Recorded Self-Assessment Examination 2020

A. DEXA scan
B. Brace treatment with a Jewett hyperextension brace
C. Anterior lumbar corpectomy and arthrodesis with instrumentation
D. Posterior lumbar decompression and fusion

R: A
The patient has MRI findings throughout her lumbar spine consistent with old compression
fractures. Given the imaging findings and advanced age, she is at high risk for osteoporosis
and subsequent fragility fractures. Management should consist of a DEXA scan to evaluate
her degree of osteoporosis and begin medical treatment as appropriate. Because acute
fracture is unlikely, and she has no neurologic compromise, neither bracing nor surgical
treatment is indicated.

Question 86 of 100
A 22-year-old man is involved in a motor vehicle crash. He is neurologically intact.
His CT scan and MRI scans, respectively, are shown in Figures 1 through 3. The
decision is made to proceed with surgery. When comparing fusion versus pedicle
screw fixation without fusion, you advise the patient that

A. pedicle screw fixation alone is associated with less blood loss and shorter operating room
time.
B. the addition of a fusion results in improved long-term outcomes.
C. if a fusion is not performed, the pedicle screws will need to be removed in 1 year.
D. pedicle screw fixation alone is likely to result in increased kyphosis at the fracture site.

R: A
The patient has a burst fracture of the spine. The use of percutaneous pedicle screws
without fusion has been shown to result in less blood loss and decreased operating room
time. It has been shown to produce equivalent outcomes compared with fusions and has
not been associated with increased kyphotic deformity. Although the screws can be removed
once the fracture has healed, this is not necessary if the patient is asymptomatic.
Adult Spine Scored and Recorded Self-Assessment Examination 2020

Question 87 of 100
A 29-year-old woman is seen in the emergency department with a 24-hour history of
severe back and leg pain precipitated by weight-lifting. The patient reports bilateral
leg pain and is unable to urinate. She has dense anesthesia in the perineal region on
examination. A MRI scan is shown in Figure 1. The patient is taken to surgery urgently.
What is her prognosis for recovery?

A. Improvement in her pain and sensory symptoms following surgery but may have residual
bladder dysfunction
B. Decreased pain following surgery; sensory deficits and bladder function are not likely to
improve
C. Complete resolution of pain and will have normal sensation and bladder function following
surgery
D. Improvement in her pain and complete return of bladder function following surgery;
sensation may not return

R: A
The patient with cauda equina syndrome should be taken to surgery urgently to provide the
best chance of symptom resolution. However, many studies indicate that patients with cauda
equina syndrome do not return to a completely normal status even following urgent surgery.
Whereas pain is typically relieved after surgery, other deficits, especially bladder and sexual
dysfunction, may persist. Particularly in light of the patient's severe saddle anesthesia, she
may have a poor prognosis for recovery of normal bladder function.

Question 88 of 100
The risk for perioperative venous thromboembolism (VTE) during spine surgery is
most associated with

A. operative time.
B. preoperative ambulatory status.
C. BMI.
Adult Spine Scored and Recorded Self-Assessment Examination 2020

D. diabetes.

R: B
In 2016, Wang and associates performed a meta-analysis of 12 retrospective studies and
reported an overall incidence of perioperative VTE of 2% in patients following spine surgery.
The following were found to have increased risk for perioperative VTE: preoperative poor
ambulatory status (odds ratio 4.8), diabetes (odds ratio 2.12), and hypertension (odds ratio
1.59). In contrast, surgical time, age, BMI, smoking, and specific surgical procedure were
not associated with increased risk of perioperative VTE. In another study, Piper and
associates used data (22,434 spine surgeries) from the American College of Surgeons
National Surgical Quality Improvement Program database from 2006 to 2010. Nine patient-
specific risk factors were associated with VTE, including hypertension (odds ratio 2.08),
dependent functional status (odds ratio 4.34), malignancy (odds ratio 6.83), inpatient status
(odds ratio 7.13), paraplegia (odds ratio 3.74), and quadriplegia (odds ratio 5.63).

Question 89 of 100
Figures 1 and 2 are the CT and MR spine images of an 82-year-old man who has a
history of ankylosing spondylitis falls onto his back. He has no neurologic deficits
upon examination in the emergency department. What is the most appropriate next
step?

A. Obtain upright radiographs


B. Thoracolumbar orthosis
C. Posterior stabilization and fusion
D. Laminectomy

R:C
Spinal fractures in patients with ankylosing spondylitis are unstable and generally
necessitate surgical intervention. In a patient with a spinal fracture in the setting of
Adult Spine Scored and Recorded Self-Assessment Examination 2020

ankylosing spondylitis, posterior instrumented fusion is an appropriate surgical procedure.


Treatment with a thoracolumbar orthosis is not an option for patients with extension
distraction injuries in the setting of an ankylosed spine because of risk for displacement.
Similarly, simply checking upright radiographs is generally not advocated. Laminectomy
alone is inappropriate for this patient because there is no cord compression and neurologic
symptoms are absent. Stabilization is the treatment goal.

Question 90 of 100
Compared with iliac crest bone graft, rh2 bone morphogenetic protein (BMP) has a
higher rate of

A. reoperation after lumbar fusion.


B. death when used in the anterior cervical spine.
C. cost efficacy when the cost of implants and cost of reoperation are considered.
D. pseudarthrosis in lumbar fusion.

R: B
BMP is contraindicated for use in the anterior cervical spine. The U.S. Food and Drug
Administration produced a warning to not use BMP in anterior cervical surgery due to a
higher risk of death (secondary to soft-tissue swelling). When it is used in the lumbar spine,
BMP results in lower reoperation rates, higher fusion, and greater cost efficacy when the
cost of implants and reoperation are considered.
Question 91 of 100
Figure 1 depicts the cervical MRI from a 40-year-old woman with a 1-month history of
neck pain, neck stiffness, and electric-like right arm pain with certain neck
movements. She has tried anti-inflammatory medication for the pain. On physical
examination, she has a normal tandem gait, her motor and sensory functions are
intact, and she has normal reflexes. She displays a positive Spurling sign. The patient
states that she can relieve her symptoms temporarily by raising her right arm over
her head. What is the best next step?

A. Referral to physical therapy


B. Prescription for immediate release opioids
Adult Spine Scored and Recorded Self-Assessment Examination 2020

C. Scheduling of an elective anterior cervical discectomy and fusion


D. Scheduling of an elective cervical total disk arthroplasty

R: A
This patient has a cervical disk herniation with symptomatic radiculopathy. The herniation is
still in the acute phase, and the patient gives no account of clinical progression or worsening.
Generally, the natural history of cervical radiculopathy is favorable, with resolution in most
cases. Nonsurgical management remains a reasonable treatment option at this point. The
use of opioid medications carries the risk of addiction or abuse; therefore, prescribing
opioids at this time is not desirable. Because she does not have myelopathy or rapidly
progressive neurologic symptoms, surgical treatment also is not advisable at this time. The
onset of symptoms remains acute at only 4 weeks.

Question 92 of 100
A 36-year-old man has a 2-day history of acute lower back pain with severe radicular
symptoms in the left lower extremity. The patient has a positive straight leg test at 40
degrees on the left side and mild decreased sensation on the dorsum of the left foot.
What is the most appropriate management at this time?

A. Urgent admission to the hospital for surgical intervention


B. Immediate MRI of the lumbar spine as an outpatient
C. Anti-inflammatory medications and activity modification
D. Caudal epidural steroid injection

R: C
In the absence of any severe progressive neurologic deficits or other red flags, the most
appropriate management for an acute lumbar disk herniation is nonsurgical care.
Nonsurgical treatments such as limited bed rest, anti-inflammatory medications, and
judicious use of pain medications are appropriate in this clinical situation. Up to 90% of
patients will experience a resolution of symptoms without the need for surgical intervention
within a 3-month window. In the acute setting, with no neurologic deficits, immediate MRI of
the lumbar spine is neither beneficial nor warranted. Likewise, without signs of an acute
deficit, emergent surgical intervention and caudal epidural steroid injections are not needed.

Question 93 of 100
Figure 1 is the MRI scan of a 68-year-old woman who fell out of a second story
building and sustained a U-shaped sacral fracture. She is neurologically intact and
has no other major injuries. The patient is offered the option of either lumbopelvic
fixation or iliosacral fixation of her U-shaped sacral fracture. The patient inquires as
to the advantage lumbopelvic as compared with iliosacral screw fixation. She should
be told that lumbopelvic fixation
Adult Spine Scored and Recorded Self-Assessment Examination 2020

A. and stand-alone iliosacral screw fixation are equivalent.


B. allows for immediate weight bearing and increased likelihood of discharge to home.
C. has less operative time compared with iliosacral screw fixation.
D. has less blood loss compared with iliosacral screw fixation.

R: B
Lumbopelvic fixation (pedicle screws, iliac screws) has more stability than a stand-alone
iliosacral screw. The initial description of iliosacral screw fixation of U-shaped sacral fracture
by Nork and associates recommended non-weight bearing for 2 months and use of thoracic
lumbosacral hip orthosis for 6 to 8 weeks. In a comparison of lumbopelvic fixation with
iliosacral screws, Kelly and associates demonstrated that lumbopelvic fixation allowed
immediate weight bearing and increased likelihood of discharge to home; however, iliosacral
screw fixation led to a shorter operative time and less blood loss.

Question 94 of 100
Which of the angles depicted in Figure 1 has been found to most closely correlate
with a patient's lumbar lordosis, thoracic kyphosis, and overall sagittal alignment?

A. A
B. B
Adult Spine Scored and Recorded Self-Assessment Examination 2020

C. C
D. D

R: A
Angle A represents pelvic incidence (PI), a constant anatomic relationship between the
pelvis and sacrum. Angle B represents pelvic tilt, and angle C represents sacral slope. Pelvic
tilt and sacral slope can change depending on the rotation of the pelvis. Pelvic incidence
has been found to directly correlate with the magnitude of lumbar lordosis and thoracic
kyphosis because it determines the angle at the base of the spine (the lumbosacral junction).
To obtain sagittal balance, the remainder of the spine compensates, resulting in the degree
of lumbar lordosis and thoracic kyphosis to maintain an upright posture. Thus, PI must be
considered in the evaluation of sagittal balance and potential reconstructive procedures.
Angle D represents the T1 angle.

Question 95 of 100
Which injury has been shown to have the greatest impact on quality of life in a 35-
year-old man with a traumatic sacral fracture?

A. Presence of an iliac artery injury


B. Neurological injury
C. Associated unstable pelvic ring fracture
D. Soft tissue degloving injury

R: B
Traumatic sacral fractures in younger patients are often high-energy injuries. They are
commonly associated with other injuries, including fractures of the pelvic ring and long bones.
Because of their close anatomic location, sacral fractures are commonly associated with
injuries to the iliac vessels. Significant soft-tissue injuries, including fascial degloving injuries
are not uncommon. Neurological injuries can occur in up to 25% of patients with sacral
fractures. They can range from nerve root injuries to cauda equina syndrome. The presence
and severity of a neurological injury has been shown to have the greatest impact on quality
of life following these injuries.

Question 96 of 100
Figures 1 through 3 are the preoperative radiographs and a T2-weighted MR image of
a patient treated with surgery for spondylolisthesis and neuroforaminal stenosis.
Figure 4 is the postsurgical radiograph. Interbody fusion offers which advantage over
posterolateral fusion (PLF)?
Adult Spine Scored and Recorded Self-Assessment Examination 2020

A. Decreased blood loss


B. Less neural retraction
C. Improved neuroforaminal height
D. Lower fusion rates

R: C
Interbody fusion, when compared to PLF, is a predictor of more substantial blood loss.
Multilevel posterior lumbar interbody fusion (PLIF) is an independent predictor of blood loss
for posterior spine fusion. Some retrospective studies suggest that fusion rates are higher
for transforaminal lumbar interbody fusion (TLIF) than PLF, but this finding has not been
borne out in prospective studies. The main advantage of TLIF in the context of this question
is restoration of neuroforaminal height, and many surgeons will consider TLIF or PLIF for
that reason. The parasagittal MR image seen in Figure 3 shows neuroforaminal narrowing.
The pre- and postsurgical radiographs show a difference in neuroforaminal height.

Question 97 of 100
On examination, a clinician finds that a patient has difficulty with grip and release,
loss of motor strength, sensory changes, intrinsic wasting, the finger escape sign,
and spasticity. These findings are best described as

A. myelopathic hand.
B. rheumatoid arthritis of the hand.
C. carpal tunnel hand.
D. brachial plexopathy.

R: A
Myelopathic hand is a term used to describe a patient with myelopathy and myelopathic
findings in the hand. Typical myelopathic symptoms include upper motor findings, including
difficulty with hand dexterity, hyperreflexia, a positive Hoffman sign, spasticity, a positive
Romberg sign, and gait changes/ataxia.

Question 98 of 100
Adult Spine Scored and Recorded Self-Assessment Examination 2020

Figures 1 through 3 show the radiograph and CT images of a 68-year-old woman who
sustained a cervical injury after tripping over her cat. She has mild facial trauma,
which includes a broken upper incisor and a nonsurgical nasal fracture. She is
neurologically intact. Her past medical history is consistent with obstructive sleep
apnea, non–insulin-dependent diabetes mellitus (hemoglobin A1c level of 9.0), and
morbid obesity, with a body mass index of 40. What is the preferred treatment for this
patient?

A. Management with a hard collar


B. Halo vest immobilization
C. Posterior C1-2 arthrodesis
D. Anterior odontoid screw fixation

R: C
Posterior C1-2 fusion with instrumentation provides stability and pain relief with excellent
clinical outcomes despite the loss of C1-2 motion. Hard collar immobilization and halo vest
immobilization both carry a substantial risk of nonunion in this patient because of her age,
fracture displacement, residual fracture gap, and medical condition. Anterior odontoid screw
fixation theoretically preserves C1-2 motion. In this case, the fracture is not reduced.
Concentric reduction is a requisite for osteosynthesis of the odontoid. Her body habitus also
may not allow anterior odontoid fixation.

Question 99 of 100
A 56-year-old man is brought to the emergency department by paramedics following
a high-speed motor vehicle collision. He has obvious head trauma as seen by bilateral
periorbital ecchymoses, substantial facial swelling, and a large bitemporal scalp
laceration. He is not alert, but he is responsive to painful stimuli, and he moves all
four extremities. What radiographic test would you order first to assess his cervical
spine for potential injury?

A. A cross-table lateral cervical spine plain radiograph


B. A complete cervical spine series
C. CT of the cervical spine
D. Cervical MRI

R: C
Adult Spine Scored and Recorded Self-Assessment Examination 2020

CT of the cervical spine is fast and readily available in most centers. The reported sensitivity
of CT is greater than 95%, whereas specificity is almost 100%. In contrast, plain radiographs
have a sensitivity of 70% and a missed injury rate of 15% to 30%. CT also has been found
to be as cost effective or more cost effective compared with plain radiographs in diagnosing
cervical injuries. MRI is expensive, not always readily available, and inferior to CT in
diagnosing bony injuries. In this patient, dynamic imaging in the form of flexion-extension
views is contraindicated as a first line radiographic test. The patient may have an unstable
cervical injury which could be exacerbated with motion. The patient's mental status also
does not allow voluntary motion. The maneuver would have to be done by the physician or
radiology technician.

Question 100 of 100


A 65-year-old otherwise healthy woman has had 7 months of progressive low back
pain that prevents her from sleeping at night. Her MRI scan is shown in Figures 1 and
2. She has had a full staging workup, which has not revealed any other masses. A CT-
guided biopsy was obtained and the results are shown in Figure 3. What is the most
appropriate treatment of this tumor?

A. Intralesional curettage
B. Palliative radiation
C. En bloc resection
D. Chemotherapy

R: C
The patient has a chordoma. The physaliferous cells in the histologic figure confirm the
diagnosis. The two most common primary tumors of the spine are chordoma and
chondrosarcoma. In both cases, the literature supports en bloc resection. Curettage is
associated with high local recurrence rates and should be discouraged as a stand-alone
treatment. Palliative radiation is not known to be effective because the radiation dose is <50
Gy. There are no effective chemotherapies for either chordoma or chondrosarcoma.

You might also like